Mental Health Exam 4 CH 16, 17, 22, 24, 26 & 27

अब Quizwiz के साथ अपने होमवर्क और परीक्षाओं को एस करें!

Which neurotransmitter imbalance has been shown to be related to impulsive aggression? A. Low levels of ã-aminobutyric acid B. Low levels of serotonin C. High levels of dopamine D. High levels of acetylcholine

B. Low levels of serotonin Low serotonin levels have been implicated in several research studies as being a factor in impulsive aggression.REF: 516

A nurse is caring for a client who is speaking in a loud voice with clenched fists. Which of the following actions should the nurse take? A. Insist that the client stop yelling B. Request that other staff members remain close by C. Move as close to the client as possible D. Walk away from the client

B. Request that other staff members remain close by

Lucas is a nurse on a medical floor caring for Kelly, a 48-year-old patient with newly diagnosed type 2 diabetes. He realizes that depression is a complicating factor in the patient's adjustment to her new diagnosis. What problem has the most potential to arise? A. Development of agoraphobia B. Treatment nonadherence C. Frequent hypoglycemic reactions D. Sleeping rather than checking blood sugar

B. Treatment nonadherence

You are caring for a 67-year-old patient who has been receiving hemodialysis for three months. Yolanda reports that she feels angry whenever it is time for her dialysis treatment. You attribute this to: A. Organic changes in Yolanda's brain B. A flaw in Yolanda's personality C. A normal response to grief and loss D. Denial of the reality of a poor prognosis

C. A normal response to grief and loss

The nurse is caring for a depressed patient who was rescued from an earthquake. As part of the counseling and treatment, the nurse wants to assess the patient's family and the patient's stressors. What appropriate action will the nurse take? 1. Interact with the patient's family. 2. Check the patient's medical history. 3. Ask the patient to describe his or her feelings. 4. Ask the patient what makes him or her feel better.

3. Ask the patient to describe his or her feelings.

What precipitating emotional factor has been associated with an increased incidence of cancers? select all that apply A. Anxiety B. Job related stress C. Acute grief D. Feelings of hopelessness and despair from depression E. Prolonged, intense stress

D. Feelings of hopelessness and despair from depression E. Prolonged, intense stress

Parents express concern when their 5-year-old child, who is receiving treatment for cancer, keeps referring to an imaginary friend, Candy. The nurse explains that: 1. Children this age usually have imaginary friends 2. The child needs more of their one-on-one attention 3. The imaginary friend is a coping mechanism the child is using 4. It is nothing to worry about unless the child starts to isolate socially

3. The imaginary friend is a coping mechanism the child is using

The nurse is caring for a patient with rape-trauma syndrome. What outcome shown by the patient after 8 weeks indicates effective crisis intervention? 1. The patient maintains good hygiene. 2. The patient sleeps for a short span of time. 3. The patient concentrates on the given task. 4. The patient uses nonverbal communication.

3. The patient concentrates on the given task.

When does a patient have symptoms of self-hatred and depression? 1. When the patient has acute confusion 2. When the patient has post-trauma syndrome 3. When the patient has chronic low self-esteem 4. When the patient has impaired social interaction

3. When the patient has chronic low self-esteem

15. Select the most therapeutic manner for a nurse working with a patient beginning treatment for alcohol addiction. a. Empathetic, supportive b. Skeptical, guarded c. Cool, distant d. Confrontational

ANS: A Support and empathy assist the patient to feel safe enough to start looking at problems. Counseling during the early stage of treatment needs to be direct, open, and honest. The other approaches will increase patient anxiety and cause the patient to cling to defenses.

6. John Patrick is a widower with four daughters. He has enjoyed a healthy relationship with all of them until they reached puberty. As each girl began to mature physically, he acted in an aggressive manner, beating her without provocation. John Patrick is most likely acting on: a. Self-protective measures b. Stress of raising four daughters c. Frustration of unhealthy desire d. Motivating his daughters to be chaste

c. Frustration of unhealthy desire

When discussing the symptoms of posttraumatic stress disorder (PTSD), the nurse correctly states: 1. "The symptoms can occur almost immediately or can take years to manifest." 2. "PTSD causes agitation and hypervigilance, but rarely chronic depression." 3. "PTSD is an emotional response that does not cause significant changes in brain chemistry." 4. "When experiencing a flashback, the patient generally experiences a slowing of responses."

1. "The symptoms can occur almost immediately or can take years to manifest."

The expected outcome at the conclusion of crisis intervention therapy is that the patient will function in which of the following ways? 1. At the pre-crisis level 2. At a higher level than before the crisis 3. Only marginally below the pre-crisis level 4. Without aid from identified support systems

1. At the pre-crisis level

Which event leads to the development of a situational crisis in patients? 1. Loss of a job 2. Birth of a child 3. Natural disaster 4. Physical assault

1. Loss of a job

When a toddler's mother is hospitalized for several months, the nursing diagnosis risk for impaired parent/child attachment is related to prolonged separation, which has been included in the child's plan of care. The most appropriate outcome would be that: 1. Mother and child show signs of healthy bonding 2. The father is able to assume the mother's role in her absence 3. The mother is discharged and returned home as soon as possible 4. The child is able to transfer nurturing needs to another available adult

1. Mother and child show signs of healthy bonding

What is the priority concern of the crisis intervention nurse? 1. Patient safety 2. Setting up future contacts 3. Brainstorming possible solutions 4. Working through termination issues

1. Patient safety

After the bomb explosions at the Boston Marathon in 2013, members of the community donned t-shirts that said, "Boston Strong." Which phase of the disaster management continuum is evident by this behavior? 1. Recovery 2. Mitigation 3. Evaluation 4. Preparedness

1. Recovery

An adult has a family history of colon cancer, so this adult has screening colonoscopies every 5 years. At age 55, the colonoscopy reveals a malignant tumor in the ascending colon. The nurse should assess this adult for which type of crisis? 1. Situational 2. Adventitious 3. Maturational 4. Developmental

1. Situational

Which type of crisis arises from events that are extraordinary, external rather than internal, and often unanticipated? 1. Situational 2. Maturational 3. Adventitious 4. Posttraumatic stress

1. Situational

An individual comes to the crisis clinic for a first visit and reports not sleeping well, anxiety, and excessive crying. These symptoms occurred shortly after a sudden job loss and losing his or her home to a tornado that devastated the town in which the patient lives. Which of the following statements regarding crisis accurately describes the individual's situation? 1. The patient is experiencing both a situational and an adventitious crisis. 2. The patient is experiencing a situational crisis with the added stress of financial burden. 3. The patient is experiencing ineffective coping and should be hospitalized for intensive therapy. 4. The patient is experiencing low self-esteem from the job loss, as well as anger because of the loss of his or her home.

1. The patient is experiencing both a situational and an adventitious crisis.

What is the best description of a psychiatric advance directive plan? 1. Outpatient care with the goal of preventing hospitalization 2. Proactive method of addressing a crisis situation before it occurs 3. Legal, written statement regarding a person's wishes regarding medical treatment 4. Continuous 24-hour observation and supervision for those not requiring inpatient services

2. Proactive method of addressing a crisis situation before it occurs

A family's home and possessions are lost when a massive forest fire burns out of control. What type of crisis has occurred? 1. Situational 2. Adventitious 3. Maturational 4. Developmental

2. Adventitious

Whose model does a nurse follow while conducting crisis intervention for a patient with acute stress disorder? 1. Gerald Caplan 2. Albert R. Roberts 3. Erich Lindemann 4. Donna Aguilera and Janice Mesnic

2. Albert R. Roberts

Which government entity is charged with coordinating responses to American disasters, particularly in situations in which local and state resources are inadequate to the presenting challenge? 1. Department of Defense (DOD) 2. Department of Homeland Security (DHS) 3. Department of Health and Human Services (DHHS) 4. Department of Housing and Urban Development (HUD)

2. Department of Homeland Security (DHS)

Which outcome indicates the individual is demonstrating a commonly observed but negative coping strategy after a crisis event? Select all that apply. 1. Running 5 miles daily 2. Gaining 10 pounds over a 6-week period 3. Losing one's driver's license for driving drunk 4. Offering numerous excuses for not socializing 5. Scheduling spiritual counseling sessions three times a week

2. Gaining 10 pounds over a 6-week period 3. Losing one's driver's license for driving drunk 4. Offering numerous excuses for not socializing

Which behavior best supports the diagnosis of posttraumatic stress disorder (PTSD) in a 4-year-old child? 1. Overeating 2. Hypervigilance 3. A drive to be perfect 4. Passivity

2. Hypervigilance

The nurse interacts with a patient at a crisis center. The patient speaks of distressing events and makes dismissive comments. What diagnosis should the nurse make from the patient's symptoms? 1. Acute confusion 2. Ineffective denial 3. Impaired social interaction 4. Disturbed personal identity

2. Ineffective denial

A child who is able to regain mental stability after a traumatic event is said to be: 1. Mature. 2. Resilient. 3. Autonomous. 4. Independent.

2. Resilient.

A nurse is conducting crisis intervention for a patient who is a victim of physical assault and learns that the patient is confused and overwhelmed. Which strategy does the nurse implement while interacting with the patient to make the patient feel comfortable? 1. The nurse avoids making eye contact with the patient. 2. The nurse summarizes the information given by the patient. 3. The nurse advises the patient to have regular follow-up visits. 4. The nurse refrains from giving frequent feedback to the patient.

2. The nurse summarizes the information given by the patient.

An adult recently diagnosed with multiple sclerosis says, "I'm worried I won't be able to support my family or send my children to college." This person begins drinking alcohol heavily and omitting prescribed medications. What is the likely cause of the patient's behavior? 1. The patient is in a state of equilibrium.An adult recently diagnosed with multiple sclerosis says, "I'm worried I won't be able to support my family or send my children to college." This person begins drinking alcohol heavily and omitting prescribed medications. What is the likely cause of the patient's behavior? 1. The patient is in a state of equilibrium. 2. The patient is in a state of situational crisis. 3. The patient is reflecting on the situational event. 4. The patient is perceiving the event in a distorted way.

2. The patient is in a state of situational crisis.

Which statement concerning syndromes seen in other cultures, such as piblokto, Navajo frenzy witchcraft, and amok, is true? 1. They are physical disorders, not mental disorders. 2. They are culture-bound syndromes that are not dissociative disorders. 3. They are dissociative disorders, such as dissociative identity disorders. 4. They are myths or rumors, because they have not been sufficiently studied to be classified as real.

2. They are culture-bound syndromes that are not dissociative disorders.

After a major hurricane destroys a community, which statement best indicates that an individual is likely to maintain or promptly return to a state of equilibrium? 1. "This storm wasn't so bad. It could have killed more people or destroyed the water and sewer lines." 2. "I think we'll be fine. We're getting plenty of support and assistance from other communities." 3. "I've been through big storms before. If we pull together, we can help each other and rebuild our community." 4. "When my parent died 8 years ago, I got so depressed I was unable to care for my children or return to work."

3. "I've been through big storms before. If we pull together, we can help each other and rebuild our community."

A man was killed during a robbery 10 days ago. His widow, who has a long history of mental illness, cries spontaneously when talking to the nurse about her loss. What is the nurse's most therapeutic response? 1. "This loss is harder to accept because you have mental illness. Try to focus on other activities." 2. "I'm concerned that you are crying so much. Your grief over your husband's death has gone on too long." 3. "The sudden death of your husband is hard to accept. I am glad you are able to tell me about how you are feeling." 4. "Your tears let me know you are not coping appropriately with your loss. Let's make an appointment with your health care provider."

3. "The sudden death of your husband is hard to accept. I am glad you are able to tell me about how you are feeling."

A major hurricane destroyed a community. Afterward, which comment best predicts that a victim will be able to maintain or promptly return to a state of equilibrium? 1. "I think we will be fine. There are emergency agencies that will take care of us." 2. "It seems that trouble always finds me. Like in the past, I will do my best to cope with this disaster." 3. "We had had big storms before. If we pull together, we can help each other and rebuild our community." 4. "This storm wasn't so bad. It could have killed more people or destroyed the underground power lines."

3. "We had had big storms before. If we pull together, we can help each other and rebuild our community."

A crisis is so acutely uncomfortable to the individual that it is likely to self-resolve in what time frame? 1. 1 to 10 days 2. 1 to 3 weeks 3. 4 to 6 weeks 4. 3 to 4 months

3. 4 to 6 weeks

Which child is at greatest risk for developing attachment problems as a result of a neurobiological development? 1. A 7-year-old male 2. A 13-year-old male 3. A 4-year-old female 4. A 10-year-old female

3. A 4-year-old female

Which best describes the principle of crisis intervention? 1. Support that includes all people who are available that can be depended upon during the time of crisis 2. An action that promotes mental health and reduces mental illness to decrease the incidence of crisis 3. A directive, time-limited, and goal-directed strategy designed to assist individuals who are experiencing crisis 4. A technique in which those affected by a crisis or disaster come together to share thoughts, feelings, and experiences

3. A directive, time-limited, and goal-directed strategy designed to assist individuals who are experiencing crisis

When a tornado results in the loss of homes, businesses, and life, the town residents are likely to experience what type of crisis? 1. Situational crisis 2. Maturational crisis 3. Adventitious crisis 4. Endogenous crisis

3. Adventitious crisis

According to attachment theory, relationship disorders are related to trauma associated with: 1. Culture or religion 2. Siblings or strangers 3. Caregivers or parents 4. Insufficient food or shelter

3. Caregivers or parents

What is the primary factor that makes an event a crisis? 1. Type of loss that the event causes. 2. Nature of the stress caused by the event. 3. Ineffectiveness of one's usual coping strategies. 4. Preexistence of a stress-related mental illness.

3. Ineffectiveness of one's usual coping strategies.

When the patient begins to sob uncontrollably and speech becomes so incoherent that the patient is unable to give the nurse any information, the immediate interventions will focus on which of the following? 1. Securing hospital admission 2. Contacting a family member or close friend 3. Lowering the patient's anxiety level from severe to moderate 4. Assisting the patient to identify new effective coping strategies

3. Lowering the patient's anxiety level from severe to moderate

Mason and Charlie, both 16 years old, were involved in a bad car accident in which they were both passengers. Mason spoke with a counselor about the incident once and has been able to move forward with little dysfunction. Charlie has been experiencing anxiety and an inability to concentrate in school even after numerous counseling sessions. What is the most likely reason that accounts for the difference in the way individuals who experience the same event are affected emotionally by that event? 1. Previous trauma 2. Individual personality 3. Perception of the event 4. The type of counseling provided

3. Perception of the event

A patient is treated in the emergency department for injuries sustained while vacationing hundreds of miles away from home. To best meet the patient's emotional needs, the nurse should do which of the following? 1. Arrange to hospitalize the patient. 2. Refer the patient for traditional psychotherapy for post-traumatic stress disorder. 3. Provide temporary support by arranging shelter and contacting the patient's friends. 4. Suggest that contacting a victim support group would be more appropriate than crisis intervention.

3. Provide temporary support by arranging shelter and contacting the patient's friends.

A patient says to the nurse, "Sometimes I feel like I'm floating above my body, watching it from the outside." When documenting this observation, which term applies? 1. Fugue 2. Amnesia 3. Derealization 4. Depersonalization

4. Depersonalization

Which statement about dissociative disorders is true? 1. Dissociative symptoms are always negative. 2. Dissociative symptoms are usually a cry for attention. 3. Dissociative symptoms are under the person's conscious control. 4. Dissociative symptoms are not under the person's conscious control.

4. Dissociative symptoms are not under the person's conscious control.

The nurse is counseling a patient at a crisis center. After assessing the patient's situation, the nurse identifies that the patient is suffering from adventitious crisis. Which patient response supports the nurse's findings? 1. "I have nothing to do and feel worthless." 2. "I love my parents. Why did they leave me alone?" 3. "My husband assaults me every day; I want to put an end to my life." 4. "Terrorists brutally killed all the passengers; I somehow managed to escape."

4. "Terrorists brutally killed all the passengers; I somehow managed to escape."

To assess the patient's perception of the event precipitating a crisis, the nurse would initially ask which of the following questions? 1. "Who is available to help you?" 2. "Can you give me the name of someone you trust?" 3. "During difficult times in the past, what has helped you?" 4. "What was happening just before you began feeling this way?"

4. "What was happening just before you began feeling this way?"

A nurse conducts an initial interview with a veteran of two tours in the war with Afghanistan. The veteran says, "Sometimes I still hear explosions but I know I am safe in my home." What is the nurse's best response? 1. "Your description of flashbacks commonly is associated with acute stress disorder. You need to have additional treatment." 2. "Exposure to intermittent explosive devices often damages a person's ears. Let's arrange for some tests of your hearing and balance." 3. "Your experience in the war ended over two years ago. It is time for you to recognize that experience is over and you should move on with your life." 4. "You are describing flashbacks. These often happen after traumatic events such as war. I'd like to arrange for you to talk more about your feelings and reactions."

4. "You are describing flashbacks. These often happen after traumatic events such as war. I'd like to arrange for you to talk more about your feelings and reactions."

In the event of an adventitious crisis, which age group would be least in need of crisis intervention? 1. Children 2. Young adult 3. The elderly 4. A distinction cannot be made

4. A distinction cannot be made

The nurse is performing crisis intervention for a sexually assaulted patient. What appropriate action should the nurse take to help the patient cope with the crisis? Select all that apply. 1. Guide the patient about maintaining support systems. 2. Convince the patient to file a complaint against the attacker. 3. Encourage the patient to make conclusions about their suffering. 4. Encourage the patient to express feelings in a nondestructive manner. 5. Frequently restate the statements made by the patient during interview.

4. Encourage the patient to express feelings in a nondestructive manner. 5. Frequently restate the statements made by the patient during interview. 1. Guide the patient about maintaining support systems.

Following an assessment, the nurse concludes that the patient is in phase 2 crisis. What signs and symptoms does the nurse find in the patient? 1. Severe panic 2. Suicidal intentions 3. Intention to harm others 4. Feelings of extreme discomfort

4. Feelings of extreme discomfort

Which of the following is an example of a maturational crisis? 1. Experiencing an unplanned pregnancy 2. Losing one's job within months of retiring 3. Working at a bank that recently was robbed 4. Having one's only child leave home to attend college

4. Having one's only child leave home to attend college

Empathetic listening is therapeutic because it focuses on: 1. Reducing anxiety 2. Encouraging resilience 3. Enhancing self-esteem 4. Lessening feelings of isolation

4. Lessening feelings of isolation

A man is having difficulty adjusting to becoming a father. He has not bonded with his newborn and frequently stays out at night drinking with friends. Which type of crisis is evident? 1. External 2. Situational 3. Adventitious 4. Maturational

4. Maturational

What is the term for crises that occur as an individual moves from one developmental level to another? 1. Reactive crises 2. Situational crises 3. Adventitious crises 4. Maturational crises

4. Maturational crises

What are the two initial goals for crisis interventions? 1. Anxiety reduction and coping 2. Crisis resolution and planning 3. Assessment and nursing diagnosis 4. Patient safety and anxiety reduction

4. Patient safety and anxiety reduction

Dissociative identity disorder is characterized by: 1. The inability to recall important information 2. Recurring feelings of detachment from one's body or mental processes 3. Sudden, unexpected travel away from home and inability to remember the past 4. The existence of two or more subpersonalities, each with its own patterns of thinking

4. The existence of two or more subpersonalities, each with its own patterns of thinking

Which statement about structural dissociation of the personality is true? 1. An organic basis exists for this type of disorder. 2. Nurses perceive patients with this disorder as easy to care for. 3. No known link exists between this disorder and early childhood loss or trauma. 4. This disorder results in a split in the personality, causing a lack of integration.

4. This disorder results in a split in the personality, causing a lack of integration.

A patient diagnosed with posttraumatic stress disorder shows little symptom improvement after being prescribed a selective serotonin reuptake inhibitor (SSRI). The nurse expects that the next medication to be prescribed will be a 1. Sedative 2. Barbiturate 3. Beta blocker 4. Tricyclic antidepressant (TCA)

4. Tricyclic antidepressant (TCA)

Client Needs: Physiological Integrity 7. A rape victim says to the nurse, "I always try to be so careful. I know I should not have walked to my car alone. Was this attack my fault?" Which communication by the nurse is most therapeutic? a. Support the victim to separate issues of vulnerability from blame. b. Emphasize the importance of using a buddy system in public places. c. Reassure the victim that the outcome of the situation will be positive. d. Pose questions about the rape and help the patient explore why it happened.

A Although the victim may have made choices that made her vulnerable, she is not to blame for the rape. Correcting this distortion in thinking allows the victim to begin to restore a sense of control. This is a positive response to victimization. The distracters do not permit the victim to begin to restore a sense of control or offer use of non-therapeutic communication techniques. In this interaction, the victim needs to talk about feelings rather than prevention. PTS: 1 DIF: Cognitive Level: Apply (Application) REF: Page 556-557,559-560 TOP: Nursing Process: Implementation

Client Needs: Psychosocial Integrity 12. Which comment by a patient who recently experienced a myocardial infarction indicates use of maladaptive, ineffective coping strategies? a. "My employer should have paid for a health club membership for me." b. "My family will see me through this. It won't be easy, but I will never be alone." c. "My heart attack was no fun, but it showed me up the importance of a good diet and more exercise." d. "I accept that I have heart disease. Now I need to decide if I will be able to continue my work daily."

A Blaming someone else and rationalizing one's failure to exercise are not adaptive coping strategies. Seeing the glass as half full, using social and religious supports, and confronting one's situation are seen as more effective strategies. The distracters demonstrate effective coping associated with a serious medical condition. PTS: 1 DIF: Cognitive Level: Apply (Application) REF: Page 326-327 | Page 331 | Page 334-335 TOP: Nursing Process: Assessment

28. A nurse wants to research epidemiology, assessment techniques, and best practices regarding persons with addictions. Which resource will provide the most comprehensive information? a. Substance Abuse and Mental Health Services Administration (SAMHSA) b. Institute of Medicine - National Research Council (IOM) c. National Council of State Boards of Nursing (NCSBN) d. American Society of Addictions Medicine

ANS: A The Substance Abuse and Mental Health Services Administration (SAMHSA) is the official resource for comprehensive information regarding addictions. The other resources have relevant information, but they are not as comprehensive.

Client Needs: Psychosocial Integrity 13. A nurse assesses a patient diagnosed with conversion (functional neurological) disorder. Which comment is most likely from this patient? a. "Since my father died, I've been short of breath and had sharp pains that go down my left arm, but I think it's just indigestion." b. "I have daily problems with nausea, vomiting, and diarrhea. My skin is very dry, and I think I'm getting seriously dehydrated." c. "Sexual intercourse is painful. I pretend as if I'm asleep so I can avoid it. I think it's starting to cause problems with my marriage." d. "I get choked very easily and have trouble swallowing when I eat. I think I might have cancer of the esophagus."

A Patients with conversion (functional neurological) disorder demonstrate a lack of concern regarding the seriousness of symptoms. This lack of concern is termed "la belle indifférence." There is also a specific, identifiable cause for the development of the symptoms; in this instance, the death of a parent would precipitate stress. The distracters relate to sexual dysfunction and illness anxiety disorder. PTS: 1 DIF: Cognitive Level: Apply (Application) REF: Page 325-326 TOP: Nursing Process: Assessment

Client Needs: Physiological Integrity 20. Which assessment question could a nurse ask to help identify secondary gains associated with a somatic symptom disorder? a. "What are you unable to do now but were previously able to do?" b. "How many doctors have you seen in the last year?" c. "Who do you talk to when you're upset?" d. "Did you experience abuse as a child?"

A Secondary gains should be assessed. Secondary gains reinforce maladaptive behavior. The patient's dependency needs may be evident through losses of abilities. When secondary gains are prominent, the patient is more resistant to giving up the symptom. There may be a history of abuse or doctor shopping, but the question does not assess the associated gains. PTS: 1 DIF: Cognitive Level: Apply (Application) REF: Page 329 | Page 331 TOP: Nursing Process: Assessment

Client Needs: Physiological Integrity 19. A victim of a violent rape was treated in the emergency department. As discharge preparation begins, the victim says softly, "I will never be the same again. I can't face my friends. There is no reason to go on." Select the nurse's most appropriate response. a. "Are you thinking of harming yourself?" b. "It will take time, but you will feel the same as before the attack." c. "Your friends will understand when you explain it was not your fault." d. "You will be able to find meaning from this experience as time goes on."

A The patient's words suggest hopelessness. Whenever hopelessness is present, so is suicide risk. The nurse should directly address the possibility of suicidal ideation with the patient. The other options attempt to offer reassurance before making an assessment. PTS: 1 DIF: Cognitive Level: Analyze (Analysis) REF: Page 555 | Page 557 TOP: Nursing Process: Assessment

Client Needs: Psychosocial Integrity 12. A nurse works a rape telephone hotline. Communication with potential victims should focus on: a. explaining immediate steps victims should take. b. providing callers with a sympathetic listener. c. obtaining information for law enforcement. d. arranging counseling.

A The telephone counselor establishes where the victim is and what has happened and provides the necessary information to enable the victim to decide what steps to take immediately. Counseling is not the focus until immediate problems are resolved. The victim remains anonymous. The other distracters are inappropriate or incorrect because counselors are trained to be empathetic rather than sympathetic. PTS: 1 DIF: Cognitive Level: Understand (Comprehension) REF: Page 555-556 | Page 559 TOP: Nursing Process: Implementation

Client Needs: Psychosocial Integrity 16. A victim of a sexual assault who sits in the emergency department is rocking back and forth and repeatedly saying, "I can't believe I've been raped." This behavior is characteristic of which stage of rape-trauma syndrome? a. The acute phase reaction c. A delayed reaction b. The long-term phase d. The angry stage

A The victim's response is typical of the acute phase and shows cognitive, affective, and behavioral disruptions. This response is immediate and does not include a display of behaviors suggestive of the long-term (reorganization) phase, anger, or a delayed reaction. PTS: 1 DIF: Cognitive Level: Understand (Comprehension) REF: Page 554-555 | Page 558 (Table 29-2) TOP: Nursing Process: Assessment

Client Needs: Psychosocial Integrity 14. A patient who experienced a myocardial infarction was transferred from critical care to a step-down unit. The patient then used the call bell every 15 minutes for minor requests and complaints. Staff nurses reported feeling inadequate and unable to satisfy the patient's needs. When the nurse manager intervenes directly with this patient, which comment is most therapeutic? a. "I'm wondering if you are feeling anxious about your illness and being left alone." b. "The staff are concerned that you are not satisfied with the care you are receiving." c. "Let's talk about why you use your call light so frequently. It is a problem." d. "You frustrate the staff by calling them so often. Why are you doing that?"

A This patient is experiencing anxiety associated with a serious medical condition. Verbalization is an effective outlet for anxiety. "I'm wondering if you are anxious..." focuses on the emotions underlying the behavior rather than the behavior itself. This opening conveys the nurse's willingness to listen to the patient's feelings and an understanding of the commonly seen concern about not having a nurse always nearby as in the intensive care unit. The other options focus on the behavior or its impact on nursing and do not help the patient with her emotional needs. PTS: 1 DIF: Cognitive Level: Apply (Application) REF: Page 326-327 | Page 331-334 TOP: Nursing Process: Implementation

Client Needs: Safe, Effective Care Environment 2. Which presentations suggest the possibility of a factitious disorder, self-directed type? Select all that apply. a. History of multiple hospitalizations without findings of physical illness b. History of multiple medical procedures or exploratory surgeries c. Going from one doctor to another seeking the desired response d. Claims illness to obtain financial benefit or other incentive e. Difficulty describing symptoms

A, B Persons with factitious disorders, self-directed type, typically have a history of multiple hospitalizations and medical workups, with negative findings from workups. Sometimes they have even had multiple surgeries seeking the origin of the physical complaints. If they do not receive the desired response from a hospitalization, they may elope or accuse staff of incompetence. Such persons usually seek treatment through a consistent health care provider rather than doctor-shopping, are not motivated by financial gain or other external incentives, and present symptoms in a very detailed, plausible manner indicating considerable understanding of the disorder or presentation they are mimicking. See relationship to audience response question. PTS: 1 DIF: Cognitive Level: Understand (Comprehension) REF: Page 337-338 TOP: Nursing Process: Implementation

Client Needs: Psychosocial Integrity 3. An emergency department nurse prepares to assist with examination of a sexual assault victim. What equipment will be needed to collect and document forensic evidence? Select all that apply. a. Camera b. Body map c. DNA swabs d. Pulse oximeter e. Sphygmomanometer

A, B, C Body maps, DNA swabs, and photographs are used to collect and preserve body fluids and other forensic evidence. PTS: 1 DIF: Cognitive Level: Understand (Comprehension) REF: Page 555-556 | Page 557-558 (Box 29-1) | Page 560-561 (Nursing Care Plan 29-1) TOP: Nursing Process: Planning

Client Needs: Psychosocial Integrity 2. A patient was abducted and raped at gunpoint by an unknown assailant. Which nursing interventions are appropriate while caring for the patient in the emergency department? Select all that apply. a. Allow the patient to talk at a comfortable pace. b. Place the patient in a private room with a caregiver. c. Pose questions in nonjudgmental, empathetic ways. d. Invite the patient's family members to the examination room. e. Put an arm around the patient to demonstrate support and compassion.

A, B, C Neutral, nonjudgmental care and emotional support are critical to crisis management for the rape victim. The rape victim should have privacy but not be left alone. The rape victim's anxiety may escalate when touched by a stranger, even when the stranger is a nurse. Some rape victims prefer not to have family involved. The patient's privacy may be compromised by family presence. PTS: 1 DIF: Cognitive Level: Apply (Application) REF: Page 555-556 | Page 560-561 (Nursing Care Plan 29-1) TOP: Nursing Process: Implementation

Client Needs: Psychosocial Integrity 3. A patient diagnosed with a somatic symptom disorder says, "Why has God chosen me to be sick all the time and unable to provide for my family? The burden on my family is worse than the pain I bear." Which nursing diagnoses apply to this patient? Select all that apply. a. Spiritual distress b. Decisional conflict c. Adult failure to thrive d. Impaired social interaction e. Ineffective role performance

A, E The patient's verbalization is consistent with spiritual distress. The patient's description of being unable to provide for and burdening the family indicates ineffective role performance. No data support diagnoses of adult failure to thrive, impaired social interaction, or decisional conflict. PTS: 1 DIF: Cognitive Level: Apply (Application) REF: Page 329-333 (Table 17-3) TOP: Nursing Process: Diagnosis/Analysis

A 10-year-old child was placed in a foster home after being removed from parental contact because of abuse. The child has apprehension, tremulousness, and impaired concentration. The foster parent also reports the child has an upset stomach, urinates frequently, and does not understand what has happened. What helpful measures should the nurse suggest to the foster parents? The nurse should recommend: (select all that apply) a. conveying empathy and acknowledging the child's distress. b. explaining and reinforcing reality to avoid distortions. c. using a calm manner and low, comforting voice. d. avoiding repetition in what is said to the child. e. staying with the child until the anxiety decreases. f. minimizing opportunities for exercise and play.

A, B, C, E The childs symptoms and behavior suggest that he is exhibiting posttraumatic stress disorder. Interventions appropriate for this level of anxiety include using a calm, reassuring tone, acknowledging the childs distress, repeating content as needed when there is impaired cognitive processing and memory, providing opportunities for comforting and normalizing play and physical activities, correcting any distortion of reality, and staying with the child to increase his sense of security

The nurse interviewing a patient with suspected posttraumatic stress disorder should be alert to findings indicating the patient: (select all that apply) a. avoids people and places that arouse painful memories. b. experiences flashbacks or reexperiences the trauma. c. experiences symptoms suggestive of a heart attack. d. feels driven to repeat selected ritualistic behaviors. e. demonstrates hypervigilance or distrusts others. f. feels detached, estranged, or empty inside.

A, B, C, E, F These assessment findings are consistent with the symptoms of posttraumatic stress disorder. Ritualistic behaviors are expected in obsessive-compulsive disorder.

A young adult says, "I was sexually abused by my older brother. During those assaults, I went somewhere else in my mind. I don't remember the details. Now, I often feel numb or unreal in romantic relationships, so I just avoid them." Which disorders should the nurse suspect based on this history? Select all that apply. a. Acute stress disorder b. Depersonalization disorder c. Generalized anxiety disorder d. Posttraumatic stress disorder e. Reactive attachment disorder f. Disinhibited social engagement disorder

A, B, D Acute stress disorder, depersonalization disorder, and posttraumatic stress disorder can involve dissociative elements, such as numbing, feeling unreal, and being amnesic for traumatic events. All three disorders are also responses to acute stress or trauma, which has occurred here. The distracters are disorders not evident in this patients presentation. Generalized anxiety disorder involves extensive worrying that is disproportionate to the stressors or foci of the worrying. Reactive attachment disorder and disinhibited social engagement disorder are problems of childhood.

Client Needs: Psychosocial Integrity 4. A nurse assesses a patient suspected of having somatic symptom disorder. Which assessment findings regarding this patient support the suspected diagnosis? Select all that apply. a. Female b. Reports frequent syncope c. Rates pain as "1" on a scale of "10" d. First diagnosed with psoriasis at age 12 e. Reports insomnia often results from back pain

A, B, E There is no chronic disease to explain the symptoms for patients with somatic symptom disorder. Patients report multiple symptoms; gastrointestinal and pseudoneurological symptoms are common. This disorder is more common in women than in men. Patients with conversion disorder would have a tendency to underrate pain. PTS: 1 DIF: Cognitive Level: Apply (Application) REF: Page 325-326 TOP: Nursing Process: Assessment

Client Needs: Psychosocial Integrity MULTIPLE RESPONSE 1. When an emergency department nurse teaches a victim of rape-trauma syndrome about reactions that may occur during the long-term phase of reorganization, which symptoms should be included? Select all that apply. a. Development of fears and phobias b. Decreased motor activity c. Feelings of numbness d. Flashbacks, dreams e. Syncopal episodes

A, C, D These reactions are common to the long-term phase. Victims of rape frequently have a period of increased motor activity rather than decreased motor activity during the long-term reorganization phase. Syncopal episodes would not be expected. PTS: 1 DIF: Cognitive Level: Understand (Comprehension) REF: Page 560-561 (Nursing Care Plan 29-1) TOP: Nursing Process: Implementation

Client Needs: Psychosocial Integrity 5. A nurse's neighbor says, "I saw a news story about a man without any known illness who died suddenly after his ex-wife committed suicide. Was that a coincidence, or can emotional shock be fatal?" The nurse should respond by noting that some serious medical conditions may be complicated by emotional stress, including: (select all that apply) a. cancer. b. hip fractures. c. hypertension. d. immune disorders. e. cardiovascular disease.

A, C, D, E A number of diseases can be worsened or brought to awareness by intense emotional stress. Immune disorders can be complicated associated with detrimental effects of stress on the immune system. Others can be brought about indirectly, such as cardiovascular disease due to acute or chronic hypertension. Hip fractures are not in this group. PTS: 1 DIF: Cognitive Level: Understand (Comprehension) REF: Page 326-327 (Table 17-1) TOP: Nursing Process: Implementation

Client Needs: Safe, Effective Care Environment 4. Which aspects of assessment have priority when a nurse interviews a rape victim in an acute setting? Select all that apply. a. Coping mechanisms the patient is using b. The patient's previous sexual experiences c. The patient's history of sexually transmitted diseases d. Signs and symptoms of emotional and physical trauma e. Adequacy and availability of the patient's support system

A, D, E The nurse assesses the victim's level of anxiety, coping mechanisms, available support systems, signs and symptoms of emotional trauma, and signs and symptoms of physical trauma. The history of STDs or previous sexual experiences has little relevance. PTS: 1 DIF: Cognitive Level: Apply (Application) REF: Page 556-558 TOP: Nursing Process: Assessment

You are working on an adolescent psychiatric unit. Katy, aged 16 years, has been angry all day because her boyfriend was not allowed to visit last night. Katy is in the hallway and begins yelling, "It's not fair! You all hate me! I hate this place!" She begins pounding her fists on the wall. To deal with the situation and prevent further escalation, your best response would be to say: A. "Katy, I will help you calm down. Do you want to go to your room and talk or go to the quiet room?" B. "Katy, you may yell and bang your fists but you must do it in your own room so you don't upset the other patients." C. "Katy, stop that right now! You will not be allowed to behave like that!" D. "Katy, you will have to go into seclusion and restraints right now." a

A. "Katy, I will help you calm down. Do you want to go to your room and talk or go to the quiet room?" Approaching the patient in a calm manner and giving choices may de-escalate the situation and gives the patient some control. The patient would not be allowed to yell or possibly hurt herself alone in her room. Commands such as "stop that right now!" could further escalate the situation. Seclusion and restraint may be premature because the situation may be able to be resolved using least restrictive means.Cognitive Level: Analyze (Analysis)Nursing Process: ImplementationNCLEX: Safe and Effective Care EnvironmentText page: 520

A nurse is caring for a client who is screaming at staff members and other clients. Which of the following is a therapeutic response by the nurse to the client/ A. "Stop screaming, and walk with me outside" B. "Why are you so angry and screaming at everyone?" C. "You will not get your way by screaming" D. "What was going through your mind when you started screaming?"

A. "Stop screaming, and walk with me outside"

Andie is a patient anxiously waiting her turn to speak with you. As you are very busy, you ask Andie if she can wait a few minutes so that you can finish your task. Unfortunately the task takes longer than anticipated and you are delayed getting back to Andie. On seeing you approach her, Andie accuses you of lying and refuses to speak with you. Which response is most likely to be therapeutic at this time? A. "you are angry that I didn't speak with you when I promised I would" B. "I'm sorry for being late, but screaming at me is not the best way to handle it." C. "You are too angry to talk right now. I'll come back in 20 minutes and we can try again." D. "Why are you angry? I told you I was busy and would get to you as soon as I could."

A. "you are angry that I didn't speak with you when I promised I would"

A 23 years old is admitted with reports of abdominal pain, dizziness, and headache. When told that all the results of a physical workup have been negative, the client shares, "Now I am having back pain." Which notation in the client's medical record may alert the nurse to the possibility of malingering? A. A court date this week for drunk driving B.Was adopted at the age of 5 years C. A history of physical abuse by his stepfather D. A history of oppositional-defiant disorder E. Raised primarily in a single parent household

A. A court date this week for drunk driving Malingering is a process of fabricating an illness or exaggerating symptoms to gain a desired benefit or avoid something undesired, such as to obtain prescription medications, evade military service, or evade legal action. It is more common in men, those who have been neglected or abused in childhood, and those who have had frequent childhood hospitalizations. Adoption is not known to be a causative factor in malingering. A history of oppositional-defiant disorder is not known to a causative factor in malingering. Being raised in a single parent home is not known to be a causative factor in malingering.DIF: Cognitive Level: Analyze (Analysis)REF: page 36TOP: Nursing Process: DiagnosisMSC: NCLEX: Psychosocial Integrity

Which assessment finding is the best predictor of violence in a newly admitted client? A. A recent assault on a drinking companion B. A family history of bipolar disorder C. The nurse's subjective feeling that the client is uncooperative. D. A childhood history of being bullied at school

A. A recent assault on a drinking companion. The best predictor of violence is past episodes of violent behavior.REF: 517

Melanie is a 38-year-old female admitted to the hospital to rule out a neurological disorder. The testing was negative, yet she is reluctant to be discharged. Today she has added lower back pain and a stabbing sensation in her abdomen. The nurse suspects a factitious disorder in which Melanie may : A. Consciously be trying to maintain her role of a sick patient B. Not recognize her unmet needs to be cared for C. Protect her child from illness D. Recognize physical symptoms as a coping mechanism

A. Consciously be trying to maintain her role of a sick patient

Which intervention strategy should be avoided by staff working with a client who is shouting and flailing his arms? A. Defusing the situation by laughing or making a joke of the challenge B. Saying "Let's go to your room to talk about this" C. Moving a few staff close together as a group to provide a show of force D. Allowing one staff person to speak to the client while others provide support

A. Defusing the situation by laughing or making a joke of the challenge Ridiculing a client should always be avoided. The other options are constructive approaches to deescalation.REF: Page 520

You respond to a loud, angry voice coming from the day room, where you find Alex is pacing and shouting that he isn't "going to take his (expletive) anymore." which of the following responses is likely to be helpful in de-escalating Alex? Select all that apply A. Remain calm, quiet, and in control B. Tell Alex that his actions are unacceptable and that he must go to his room C. Match Alex's volume so that he is able to hear over his own shouting D. Ask Alex if he can tell you what is upsetting him so you may be able to help E. Sand close to Alex so you can intervene physically if needed to protect others F. Tell Alex that he could be placed in seclusion if he cannot control himself so that the patient is aware of the negative consequences

A. Remain calm, quiet, and in control D. Ask Alex if he can tell you what is upsetting him so you may be able to help

15. A patient diagnosed with borderline personality disorder was hospitalized several times after self-mutilating episodes. The patient remains impulsive. Which nursing diagnosis is the initial focus of this therapy? a. Risk for self-directed violence b. Impaired skin integrity c. Risk for injury d. Powerlessness

ANS: A Risk for self-mutilation is a nursing diagnosis relating to patient safety needs and is therefore of high priority. Impaired skin integrity and powerlessness may be appropriate foci for care but are not the priority related to this therapy. Risk for injury implies accidental injury, which is not the case for the patient with borderline personality disorder.

Which would be the most appropriate response by the nurse to help a client who is demonstrating escalating anger? A. Walk the client to his room and help him practice stress-reduction techniques, such as deep breathing or muscle relaxation B. Suggest that the client spend some time in the gym with a punching bag to relieve his stress C. Suggest that the client spend some time pacing rapidly in the hallway until he feels less stressed D. Sit with the client in the day room so that he can vent his anger and not isolate himself

A. Walk the client to his room and help him practice stress-reduction techniques, such as deep breathing or muscle relaxation In settings in which the staff can reasonably expect episodes of client anger and aggression, regular teaching and practice of verbal and nonverbal interventions are essential. The most appropriate response by the nurse would be to help the client to a quiet environment and teach or coach the client to use positive coping skills.REF: 520

The most restrictive method for dealing with an aggressive client who is out of control is A. seclusion. B. a show of force. C. verbal intervention. D. antipsychotic medication.

A. seclusion Seclusion is the most restrictive method listed, because it curtails the client's freedom of ambulation.REF: 521-522

The more a nurse's intervention is prompted by emotion A. the less likely it is to be therapeutic B. the less likely it is to be aggressive C. the more likely it is to be effective. D. the more likely it is to be empathetic.

A. the less likely it is to be therapeutic. One study reported in the text found that the nurse's response to anger from a client varied according to the interpretation given to the client's anger and to the nurse's self-appraised ability to manage the situation. Only when self-efficacy was perceived as adequate did the nurse move to help the client. When self-efficacy was not seen as adequate, nurses showed a decreased ability to process the client's message and a decreased ability to problem-solve.REF: 518-519

16. Which features should be present in a therapeutic milieu for a patient with a hallucinogen overdose? a. Simple and safe b. Active and bright c. Stimulating and colorful d. Confrontational and challenging

ANS: A Because the individual who has ingested a hallucinogen is probably experiencing feelings of unreality and altered sensory perceptions, the best environment is one that does not add to the stimulation. A simple, safe environment is a better choice than an environment with any of the characteristics listed in the other options. The other options would contribute to a "bad trip."

18. At a meeting for family members of alcoholics, a spouse says, "I did everything I could to help. I even requested sick leave when my partner was too drunk to go to work." The nurse assesses these comments as: a. codependence. b. assertiveness c. role reversal d. homeostasis.

ANS: A Codependence refers to participating in behaviors that maintain the addiction or allow it to continue without holding the user accountable for his or her actions. The other options are not supported by information given in the scenario.

13. During the third week of treatment, the spouse of a patient in a rehabilitation program for substance abuse says, "After this treatment program, I think everything will be all right." Which remark by the nurse will be most helpful to the spouse? a. "While sobriety solves some problems, new ones may emerge as one adjusts to living without drugs and alcohol." b. "It will be important for you to structure life to avoid as much stress as you can and provide social protection." c. "Addiction is a lifelong disease of self-destruction. You will need to observe your spouse's behavior carefully." d. "It is good that you are supportive of your spouse's sobriety and want to help maintain it."

ANS: A During recovery, patients identify and use alternative coping mechanisms to reduce reliance on substances. Physical adaptations must occur. Emotional responses were previously dulled by alcohol but are now fully experienced and may cause considerable anxiety. These changes inevitably have an effect on the spouse and children, who need anticipatory guidance and accurate information.

2. Which intervention is appropriate for an individual diagnosed with an antisocial personality disorder who frequently manipulates others? a. Refer requests and questions related to care to the case manager. b. Encourage the patient to discuss feelings of fear and inferiority. c. Provide negative reinforcement for acting-out behavior. d. Ignore, rather than confront, inappropriate behavior.

ANS: A Manipulative people frequently make requests of many different staff, hoping one will give in. Having one decision maker provides consistency and avoids the potential for playing one staff member against another. Positive reinforcement of appropriate behaviors is more effective than negative reinforcement. The behavior should not be ignored; judicious use of confrontation is necessary. Patients with antisocial personality disorders rarely have feelings of fear and inferiority.

11. A patient admitted to an alcoholism rehabilitation program tells the nurse, "I'm actually just a social drinker. I usually have a drink at lunch, two in the afternoon, wine with dinner, and a few drinks during the evening." The patient is using which defense mechanism? a. Denial b. Projection c. Introjection d. Rationalization

ANS: A Minimizing one's drinking is a form of denial of alcoholism. The patient is more than a social drinker. Projection involves blaming another for one's faults or problems. Rationalization involves making excuses. Introjection involves incorporating a quality of another person or group into one's own personality.

23. Select the priority nursing intervention when caring for a patient after an overdose of amphetamines. a. Monitor vital signs. b. Observe for depression. c. Awaken the patient every 15 minutes. d. Use warmers to maintain body temperature.

ANS: A Overdose of stimulants, such as amphetamines, can produce respiratory and circulatory dysfunction as well as hyperthermia. Concentration is impaired. This patient will be hypervigilant; it is not necessary to awaken the patient.

21. What is the priority intervention for a nurse beginning to work with a patient diagnosed with a schizotypal personality disorder? a. Respect the patient's need for periods of social isolation. b. Prevent the patient from violating the nurse's rights. c. Teach the patient how to select clothing for outings. d. Engage the patient in community activities.

ANS: A Patients with schizotypal personality disorder are eccentric and often display perceptual and cognitive distortions. They are suspicious of others and have considerable difficulty trusting. They become highly anxious and frightened in social situations, thus the need to respect their desire for social isolation. Teaching the patient to match clothing is not the priority intervention. Patients with schizotypal personality disorder rarely engage in behaviors that violate the nurse's rights or exploit the nurse.

30. Select the priority outcome for a patient completing the fourth alcohol-detoxification program in the past year. Prior to discharge, the patient will: a. state, "I know I need long-term treatment." b. use denial and rationalization in healthy ways. c. identify constructive outlets for expression of anger. d. develop a trusting relationship with one staff member.

ANS: A The key refers to the need for ongoing treatment after detoxification and is the best goal related to controlling relapse. The scenario does not give enough information to determine whether anger has been identified as a problem. A trusting relationship, while desirable, should have occurred earlier in treatment.

14. A patient says, "I get in trouble sometimes because I make quick decisions and act on them." Select the nurse's most therapeutic response. a. "Let's consider the advantages of being able to stop and think before acting." b. "It sounds as though you've developed some insight into your situation." c. "I bet you have some interesting stories to share about overreacting." d. "It's good that you're showing readiness for behavioral change."

ANS: A The patient is showing openness to learning techniques for impulse control. One technique is to teach the patient to stop and think before acting impulsively. The patient can then be taught to evaluate outcomes of possible actions and choose an effective action. The incorrect responses shift the encounter to a social level or are judgmental.

17. When a patient first began using alcohol, two drinks produced relaxation and drowsiness. After 1 year, four drinks are needed to achieve the same response. Why has this change occurred? a. Tolerance has developed. b. Antagonistic effects are evident. c. Metabolism of the alcohol is now delayed. d. Pharmacokinetics of the alcohol have changed.

ANS: A Tolerance refers to needing higher and higher doses of a drug to produce the desired effect. The potency of the alcohol is stable. Neither hypomagnesemia nor antagonistic effects account for this change.

9. What is the priority nursing diagnosis for a patient diagnosed with antisocial personality disorder who has made threats against staff, ripped art off the walls, and thrown objects? a. Risk for other-directed violence b. Risk for self directed violence c. Impaired social interaction d. Ineffective denial

ANS: A Violence against property, along with threats to harm staff, makes this diagnosis the priority. Patients with antisocial personality disorders have impaired social interactions, but the risk for harming others is a higher priority. They direct violence toward others; not self. When patients with antisocial personality disorders use denial, they use it effectively.

2. For which patients diagnosed with personality disorders would a family history of similar problems be most likely? Select all that apply. a. Obsessive-compulsive b. Antisocial c. Borderline d. Schizotypal e. Narcissistic

ANS: A, B, C, D Some personality disorders have evidence of genetic links, so the family history would show other members with similar traits. Heredity plays a role in schizotypal, antisocial, borderline, and obsessive-compulsive personality disorder.

33. The nurse can assist a patient to prevent substance abuse relapse by: (select all that apply) a. rehearsing techniques to handle anticipated stressful situations. b. advising the patient to accept residential treatment if relapse occurs. c. assisting the patient to identify life skills needed for effective coping. d. advising isolating self from significant others until sobriety is established. e. informing the patient of physical changes to expect as the body adapts to functioning without substances.

ANS: A, C, E Nurses can be helpful as a patient assesses needed life skills and in providing appropriate referrals. Anticipatory problem solving and role-playing are good ways of rehearsing effective strategies for handling stressful situations and helping the patient evaluate the usefulness of new strategies. The nurse can provide valuable information about physiological changes expected and ways to cope with these changes. Residential treatment is not usually necessary after relapse. Patients need the support of friends and family to establish and maintain sobriety.

23. A patient diagnosed with borderline personality disorder has self-inflicted wrist lacerations. The health care provider prescribes daily dressing changes. The nurse performing this care should: a. maintain a stern and authoritarian affect. b. provide care in a matter-of-fact manner. c. encourage the patient to express anger. d. be very rigid and challenging.

ANS: B A matter-of-fact approach does not provide the patient with positive reinforcement for self-mutilation. The goal of providing emotional consistency is supported by this approach. The distracters provide positive reinforcement of the behavior or fail to show compassion.

10. Police bring a patient to the emergency department after an automobile accident. The patient demonstrates ataxia and slurred speech. The blood alcohol level is 500 mg%. Considering the relationship between the behavior and blood alcohol level, which conclusion is most probable? The patient: a. rarely drinks alcohol. b. has a high tolerance to alcohol. c. has been treated with disulfiram (Antabuse). d. has ingested both alcohol and sedative drugs recently.

ANS: B A non-tolerant drinker would be in coma with a blood alcohol level of 500 mg%. The fact that the patient is moving and talking shows a discrepancy between blood alcohol level and expected behavior and strongly indicates that the patient's body is tolerant. If disulfiram and alcohol are ingested together, an entirely different clinical picture would result. The blood alcohol level gives no information about ingestion of other drugs.

1. A patient diagnosed with alcoholism asks, "How will Alcoholics Anonymous (AA) help me?" Select the nurse's best response. a. "The goal of AA is for members to learn controlled drinking with the support of a higher power." b. "An individual is supported by peers while striving for abstinence one day at a time." c. "You must make a commitment to permanently abstain from alcohol and other drugs." d. "You will be assigned a sponsor who will plan your treatment program."

ANS: B Admitting to being an alcoholic, making an attempt to remain alcohol-free for a day at a time, and receiving support from peers are basic aspects of AA. The other options are incorrect.

9. A patient asks for information about Alcoholics Anonymous. Select the nurse's best response. "Alcoholics Anonymous is a: a. form of group therapy led by a psychiatrist." b. self-help group for which the goal is sobriety." c. group that learns about drinking from a group leader." d. network that advocates strong punishment for drunk drivers."

ANS: B Alcoholics Anonymous (AA) is a peer support group for recovering alcoholics. Neither professional nor peer leaders are appointed.

2. A nurse reviews vital signs for a patient admitted with an injury sustained while intoxicated. The medical record shows these blood pressure and pulse readings at the times listed: 0200: 118/78 mm Hg and 72 beats/min 0400: 126/80 mm Hg and 76 beats/min 0600: 128/82 mm Hg and 72 beats/min 0800: 132/88 mm Hg and 80 beats/min 1000: 148/94 mm Hg and 96 beats/min What is the nurse's priority action? a. Force fluids. b. Consult the health care provider. c. Obtain a clean-catch urine sample. d. Place the patient in a vest-type restraint.

ANS: B Elevated pulse and blood pressure may indicate impending alcohol withdrawal and the need for medical intervention. No indication is present that the patient may have a urinary tract infection or is presently in need of restraint. Hydration will not resolve the problem.

12. What is the most challenging nursing intervention with patients diagnosed with personality disorders who use manipulation? a. Supporting behavioral change b. Maintaining consistent limits c. Monitoring suicide attempts d. Using aversive therapy

ANS: B Maintaining consistent limits is by far the most difficult intervention because of the patient's superior skills at manipulation. Supporting behavioral change and monitoring patient safety are less difficult tasks. Aversive therapy would probably not be part of the care plan because positive reinforcement strategies for acceptable behavior seem to be more effective than aversive techniques. See relationship to audience response question.

7. A patient diagnosed with borderline personality disorder has a history of self-mutilation and suicide attempts. The patient reveals feelings of depression and anger with life. Which type of medication would the nurse expect to be prescribed? a. Benzodiazepine b. Mood stabilizing medication c. Monoamine oxidase inhibitor (MAOI) d. Serotonin norepinephrine reuptake inhibitor (SNRI)

ANS: B Mood stabilizing medications have been effective for many patients with borderline personality disorder. Serotonin norepinephrine reuptake inhibitors (SNRI) or anxiolytics are not supported by data given in the scenario. MAOIs require great diligence in adherence to a restricted diet and are rarely used for patients who are impulsive.

7. A hospitalized patient diagnosed with an alcohol abuse disorder believes spiders are spinning entrapping webs in the room. The patient is fearful, agitated, and diaphoretic. Which nursing intervention is indicated? a. Check the patient every 15 minutes b. One-on-one supervision c. Keep the room dimly lit d. Force fluids

ANS: B One-on-one supervision is necessary to promote physical safety until sedation reduces the patient's feelings of terror. Checks every 15 minutes would not be sufficient to provide for safety. A dimly lit room promotes perceptual disturbances. Excessive fluid intake can cause overhydration, because fluid retention normally occurs when blood alcohol levels fall.

3. A nurse cares for a patient diagnosed with an opioid overdose. Which focused assessment has the highest priority? a. Cardiovascular b. Respiratory c. Neurologic d. Hepatic

ANS: B Opioid overdose causes respiratory depression. Respiratory depression is the primary cause of death among opioid abusers. The assessment of the other body systems is relevant but not the priority.

6. A hospitalized patient diagnosed with an alcohol abuse disorder believes the window blinds are snakes trying to get in the room. The patient is anxious, agitated, and diaphoretic. The nurse can anticipate the health care provider will prescribe a(n): a. narcotic analgesic, such as hydromorphone (Dilaudid). b. sedative, such as lorazepam (Ativan) or chlordiazepoxide (Librium). c. antipsychotic, such as olanzapine (Zyprexa) or thioridazine (Mellaril). d. monoamine oxidase inhibitor antidepressant, such as phenelzine (Nardil).

ANS: B Sedation allows for safe withdrawal from alcohol. Benzodiazepines are the drugs of choice in most regions because of their high therapeutic safety index and anticonvulsant properties.

24. A nurse set limits while interacting with a patient demonstrating behaviors associated with borderline personality disorder. The patient tells the nurse, "You used to care about me. I thought you were wonderful. Now I can see I was wrong. You're evil." This outburst can be assessed as: a. denial. b. splitting c. defensive. d. reaction formation.

ANS: B Splitting involves loving a person, then hating the person because the patient is unable to recognize that an individual can have both positive and negative qualities. Denial is unconsciously motivated refusal to believe something. Reaction formation involves unconsciously doing the opposite of a forbidden impulse. The scenario does not indicate defensiveness. See relationship to audience response question.

19. In the emergency department, a patient's vital signs are BP 66/40 mm Hg; pulse 140 beats/min; respirations 8 breaths/min and shallow. The nursing diagnosis is Ineffective breathing pattern related to depression of respiratory center secondary to narcotic intoxication. Select the priority outcome. a. The patient will demonstrate effective coping skills and identify community resources for treatment of substance abuse within 1 week of hospitalization. b. Within 4 hours, vital signs will stabilize, with BP above 90/60 mm Hg, pulse less than 100 beats/min, and respirations at or above 12 breaths/min. c. The patient will correctly describe a plan for home care and achieving a drug-free state before release from the emergency department. d. Within 6 hours, the patient's breath sounds will be clear bilaterally and throughout lung fields.

ANS: B The correct short-term outcome is the only one that relates to the patient's physical condition. It is expected that vital signs will return to normal when the CNS depression is alleviated. The patient's respirations are slow and shallow, but there is no evidence of congestion.

28. Personality traits most likely to be documented regarding a patient demonstrating characteristics of an obsessive-compulsive personality disorder are: a. affable, generous. b. perfectionist, inflexible c. suspicious, holds grudges. d. dramatic speech, impulsive.

ANS: B The individual with obsessive-compulsive personality disorder is perfectionist, rigid, preoccupied with rules and procedures, and afraid of making mistakes. The other options refer to behaviors or traits not usually associated with OCPD. See relationship to audience response question.

29. A nurse determines desired outcomes for a patient diagnosed with schizotypal personality disorder. Select the best outcome. The patient will: a. adhere willingly to unit norms. b. report decreased incidence of self-mutilative thoughts. c. demonstrate fewer attempts at splitting or manipulating staff. d. demonstrate ability to introduce self to a stranger in a social situation.

ANS: D Schizotypal individuals have poor social skills. Social situations are uncomfortable for them. It is desirable for the individual to develop the ability to meet and socialize with others. Individuals with schizotypal PD usually have no issues with adherence to unit norms, nor are they self-mutilative or manipulative.

30. A patient says, "The other nurses won't give me my medication early, but you know what it's like to be in pain and don't let your patients suffer. Could you get me my pill now? I won't tell anyone." Which response by the nurse would be most therapeutic? a. "I'm not comfortable doing that," and then ignore subsequent requests for early medication. b. "I understand that you have pain, but giving medicine too soon would not be safe." c. "I'll have to check with your doctor about that; I will get back to you after I do." d. "It would be unsafe to give the medicine early; none of us will do that."

ANS: B The patient is attempting to manipulate the nurse. Empathetic mirroring reflects back to the patient the nurse's understanding of the patient's distress or situation in a neutral manner that does not judge it and helps elicit a more positive response to the limit that is being set. The other options would not be nontherapeutic; they lack the empathetic mirroring component that tends to elicit a more positive response from the patient.

27. An adult in the emergency department states, "Everything I see appears to be waving. I am outside my body looking at myself. I think I'm losing my mind." Vital signs are slightly elevated. The nurse should suspect: a. a schizophrenic episode. b. hallucinogen ingestion. c. opium intoxication. d. cocaine overdose.

ANS: B The patient who is high on a hallucinogen often experiences synesthesia (visions in sound), depersonalization, and concerns about going "crazy." Synesthesia is not common in schizophrenia. CNS stimulant overdose more commonly involves elevated vital signs and assaultive, grandiose behaviors. Phencyclidine (PCP) use commonly causes bizarre or violent behavior, nystagmus, elevated vital signs, and repetitive jerking movements.

24. Symptoms of withdrawal from opioids for which the nurse should assess include: a. dilated pupils, tachycardia, elevated blood pressure, and elation. b. nausea, vomiting, diaphoresis, anxiety, and hyperreflexia. c. mood lability, incoordination, fever, and drowsiness. d. excessive eating, constipation, and headache.

ANS: B The symptoms of withdrawal from opioids are similar to those of alcohol withdrawal. Hyperthermia is likely to produce periods of diaphoresis.

26. A nursing diagnosis appropriate to consider for a patient diagnosed with any of the personality disorders is: a. noncompliance. b. impaired social interaction c. disturbed personal identity. d. diversional activity deficit.

ANS: B Without exception, individuals with personality disorders have problems with social interaction with others, hence, the diagnosis of "impaired social interaction." For example, some individuals are suspicious and lack trust, others are avoidant, and still others are manipulative. None of the other diagnoses are universally applicable to patients with personality disorders; each might apply to selected clinical diagnoses, but not to others.

32. A patient undergoing alcohol rehabilitation decides to begin disulfiram (Antabuse) therapy. Patient teaching should include the need to: (select all that apply) a. avoid aged cheeses. b. avoid alcohol-based skin products. c. read labels of all liquid medications. d. wear sunscreen and avoid bright sunlight. e. maintain an adequate dietary intake of sodium. f. avoid breathing fumes of paints, stains, and stripping compounds.

ANS: B, C, F The patient must avoid hidden sources of alcohol. Many liquid medications, such as cough syrups, contain small amounts of alcohol that could trigger an alcohol-disulfiram reaction. Using alcohol-based skin products such as aftershave or cologne, smelling alcohol-laden fumes, and eating foods prepared with wine, brandy, or beer may also trigger reactions. The other options do not relate to hidden sources of alcohol.

1. A nurse plans care for an individual diagnosed with antisocial personality disorder. Which characteristic behaviors will the nurse expect? Select all that apply. a. Reclusive behavior b. Callous attitude c. Perfectionism d. Aggression e. Clinginess f. Anxiety

ANS: B, D Individuals with antisocial personality disorders characteristically demonstrate manipulative, exploitative, aggressive, callous, and guilt-instilling behaviors. Individuals with antisocial personality disorders are more extroverted than reclusive, rarely show anxiety, and rarely demonstrate clinging or dependent behaviors. Individuals with antisocial personality disorders are more likely to be impulsive than to be perfectionists.

10. When a patient diagnosed with a personality disorder uses manipulation to get needs met, the staff applies limit-setting interventions. What is the correct rationale for this action? a. It provides an outlet for feelings of anger and frustration. b. It respects the patient's wishes, so assertiveness will develop. c. External controls are necessary due to failure of internal control. d. Anxiety is reduced when staff assumes responsibility for the patient's behavior.

ANS: C A lack of internal controls leads to manipulative behaviors such as lying, cheating, conning, and flattering. To protect the rights of others, external controls must be consistently maintained until the patient is able to behave appropriately.

14. The treatment team discusses the plan of care for a patient diagnosed with schizophrenia and daily cannabis abuse who is having increased hallucinations and delusions. To plan effective treatment, the team should: a. provide long-term care for the patient in a residential facility. b. withdraw the patient from cannabis, then treat the schizophrenia. c. consider each diagnosis primary and provide simultaneous treatment. d. first treat the schizophrenia, then establish goals for substance abuse treatment.

ANS: C Both diagnoses should be considered primary and receive simultaneous treatment. Comorbid disorders require longer treatment and progress is slower, but treatment may occur in the community.

25. Which characteristic of personality disorders makes it most necessary for staff to schedule frequent team meetings in order to address the patient's needs and maintain a therapeutic milieu? a. Ability to achieve true intimacy b. Flexibility and adaptability to stress c. Ability to provoke interpersonal conflict d. Inability to develop trusting relationships

ANS: C Frequent team meetings are held to counteract the effects of the patient's attempts to split staff and set them against one another, causing interpersonal conflict. Patients with personality disorders are inflexible and demonstrate maladaptive responses to stress. They are usually unable to develop true intimacy with others and are unable to develop trusting relationships. Although problems with trust may exist, it is not the characteristic that requires frequent staff meetings. See relationship to audience response question.

6. A nurse reports to the treatment team that a patient diagnosed with an antisocial personality disorder has displayed the behaviors below. This patient is detached and superficial during counseling sessions. Which behavior by the patient most clearly warrants limit setting? a. Flattering the nurse b. Lying to other patients c. Verbal abuse of another patient d. Detached superficiality during counseling

ANS: C Limits must be set in areas in which the patient's behavior affects the rights of others. Limiting verbal abuse of another patient is a priority intervention and particularly relevant when interacting with a patient diagnosed with an antisocial personality disorder. The other concerns should be addressed during therapeutic encounters.

25. A patient has smoked two packs of cigarettes daily for many years. When the patient tries to reduce smoking, anxiety, craving, poor concentration, and headache occur. This scenario describes: a. cross-tolerance. b. substance abuse c. substance addiction. d. substance intoxication.

ANS: C Nicotine meets the criteria for a "substance," the criterion for addiction is present, and withdrawal symptoms are noted with abstinence or reduction of dose. The scenario does not meet criteria for substance abuse, intoxication, or cross-tolerance.

5. Consider this comment to three different nurses by a patient diagnosed with an antisocial personality disorder, "Another nurse said you don't do your job right." Collectively, these interactions can be assessed as: a. seductive. b. detached c. manipulative. d. guilt-producing.

ANS: C Patients manipulate and control staff in various ways. By keeping staff off balance or fighting among themselves, the person with an antisocial personality disorder is left to operate as he or she pleases. Seductive behavior has sexual connotations. The patient is displaying the opposite of detached behavior. Guilt is not evident in the comments.

20. Others describe a worker as very shy and lacking in self-confidence. This worker stays in an office cubicle all day, never coming out for breaks or lunch. Which term best describes this behavior? a. Narcissistic b. Histrionic c. Avoidant d. Paranoid

ANS: C Patients with avoidant personality disorder are timid, socially uncomfortable, withdrawn, and avoid situations in which they might fail. They believe themselves to be inferior and unappealing. Individuals with histrionic personality disorder are seductive, flamboyant, shallow, and attention-seeking. Paranoia and narcissism are not evident.

16. Which statement made by a patient diagnosed with borderline personality disorder indicates the treatment plan is effective? a. "I think you are the best nurse on the unit." b. "I'm never going to get high on drugs again." c. "I felt empty and wanted to hurt myself, so I called you." d. "I hate my mother. I called her today, and she wasn't home."

ANS: C Seeking a staff member instead of impulsively self-mutilating shows an adaptive coping strategy. The incorrect responses demonstrate idealization, devaluation, and wishful thinking.

4. A patient admitted for injuries sustained while intoxicated has been hospitalized for 48 hours. The patient is now shaky, irritable, anxious, diaphoretic, and reports nightmares. The pulse rate is 130 beats/min. The patient shouts, "Bugs are crawling on my bed. I've got to get out of here." Select the most accurate assessment of this situation. The patient: a. is attempting to obtain attention by manipulating staff. b. may have sustained a head injury before admission. c. has symptoms of alcohol-withdrawal delirium. d. is having an acute psychosis.

ANS: C Symptoms of agitation, elevated pulse, and perceptual distortions indicate alcohol withdrawal delirium. The findings are inconsistent with manipulative attempts, head injury, or functional psychosis.

13. The history shows that a newly admitted patient is impulsive. The nurse would expect behavior characterized by: a. adherence to a strict moral code. b. manipulative, controlling strategies. c. acting without thought on urges or desires. d. postponing gratification to an appropriate time.

ANS: C The impulsive individual acts in haste without taking time to consider the consequences of the action. None of the other options describes impulsivity.

3. As a nurse prepares to administer medication to a patient diagnosed with a borderline personality disorder, the patient says, "Just leave it on the table. I'll take it when I finish combing my hair." What is the nurse's best response? a. Reinforce this assertive action by the patient. Leave the medication on the table as requested. b. Respond to the patient, "I'm worried that you might not take it. I'll come back later." c. Say to the patient, "I must watch you take the medication. Please take it now." d. Ask the patient, "Why don't you want to take your medication now?"

ANS: C The individual with a borderline personality disorder characteristically demonstrates manipulative, splitting, and self-destructive behaviors. Consistent limit setting is vital for the patient's safety, but also to prevent splitting other staff. "Why" questions are not therapeutic. See relationship to audience response question.

31. A nurse prepares for an initial interaction with a patient with a long history of methamphetamine abuse. Which is the nurse's best first action? a. Perform a thorough assessment of the patient. b. Verify that security services are immediately available. c. Self-assess personal attitude, values, and beliefs about this health problem. d. Obtain a face shield because oral hygiene is poor in methamphetamine abusers.

ANS: C The nurse should show compassion, care, and helpfulness for all patients, including those with addictive diseases. It is important to have a clear understanding of one's own perspective. Negative feelings may occur for the nurse; supervision is an important resource. The activities identified in the distracters occur after self-assessment.

11. One month ago, a patient diagnosed with borderline personality disorder and a history of self-mutilation began dialectical behavior therapy. Today the patient phones to say, "I feel empty and want to hurt myself." The nurse should: a. arrange for emergency inpatient hospitalization. b. send the patient to the crisis intervention unit for 8 to 12 hours. c. assist the patient to choose coping strategies for triggering situations. d. advise the patient to take an anti-anxiety medication to decrease the anxiety level.

ANS: C The patient has responded appropriately to the urge for self-harm by calling a helping individual. A component of dialectical behavior therapy is telephone access to the therapist for "coaching" during crises. The nurse can assist the patient to choose an alternative to self-mutilation. The need for a protective environment may not be necessary if the patient is able to use cognitive strategies to determine a coping strategy that will reduce the urge to mutilate. Taking a sedative and going to sleep should not be the first-line intervention because sedation may reduce the patient's ability to weigh alternatives to mutilating behavior.

27. A new psychiatric technician says, "Schizophrenia...schizotypal! What's the difference?" The nurse's response should include which information? a. A patient diagnosed with schizophrenia is not usually overtly psychotic. b. In schizotypal personality disorder, the patient remains psychotic much longer. c. With schizotypal personality disorder, the person can be made aware of misinterpretations of reality. d. Schizotypal personality disorder causes more frequent and more prolonged hospitalizations than schizophrenia.

ANS: C The patient with schizotypal personality disorder might have problems thinking, perceiving, and communicating and might have an odd, eccentric appearance; however, they can be made aware of misinterpretations and overtly psychotic symptoms are usually absent. The individual with schizophrenia is more likely to display psychotic symptoms, remain ill for longer periods, and have more frequent and prolonged hospitalizations.

4. What is an appropriate initial outcome for a patient diagnosed with a personality disorder who frequently manipulates others? The patient will: a. identify when feeling angry. b. use manipulation only to get legitimate needs met. c. acknowledge manipulative behavior when it is called to his or her attention. d. accept fulfillment of his or her requests within an hour rather than immediately.

ANS: C This is an early outcome that paves the way for later taking greater responsibility for controlling manipulative behavior. Identifying anger relates to anger and aggression control. Using manipulation to get legitimate needs is an inappropriate outcome. The patient would ideally use assertive behavior to promote need fulfillment. Accepting fulfillment of requests within an hour rather than immediately relates to impulsivity control.

35. A new patient beginning an alcoholism rehabilitation program says, "I'm just a social drinker. I usually have one drink at lunch, two in the afternoon, wine at dinner, and a few drinks during the evening." Select the nurse's most therapeutic responses. Select all that apply. a. "I see," and use interested silence. b. "I think you are drinking more than you report." c. "Social drinkers have one or two drinks, once or twice a week." d. "You describe drinking steadily throughout the day and evening." e. "Your comments show denial of the seriousness of your problem."

ANS: C, D The correct answers give information, summarize, and validate what the patient reported but are not strongly confrontational. Defenses cannot be removed until healthier coping strategies are in place. Strong confrontation does not usually take place so early in the program.

22. A patient diagnosed with borderline personality disorder self-inflicted wrist lacerations after gaining new privileges on the unit. In this case, the self-mutilation may have been due to: a. an inherited disorder that manifests itself as an incapacity to tolerate stress. b. use of projective identification and splitting to bring anxiety to manageable levels. c. a constitutional inability to regulate affect, predisposing to psychic disorganization. d. fear of abandonment associated with progress toward autonomy and independence.

ANS: D Fear of abandonment is a central theme for most patients with borderline personality disorder. This fear is often exacerbated when patients with borderline personality disorder experience success or growth.

26. Which assessment findings are likely for an individual who recently injected heroin? a. Anxiety, restlessness, paranoid delusions b. Muscle aching, dilated pupils, tachycardia c. Heightened sexuality, insomnia, euphoria d. Drowsiness, constricted pupils, slurred speech

ANS: D Heroin, an opiate, is a CNS depressant. Blood pressure, pulse, and respirations will be decreased, and attention will be impaired. The distracters describe behaviors consistent with amphetamine use, symptoms of narcotic withdrawal, and cocaine use.

19. The nurse caring for an individual demonstrating symptoms of schizotypal personality disorder would expect assessment findings to include: a. arrogant, grandiose, and a sense of self-importance. b. attention seeking, melodramatic, and flirtatious. c. impulsive, restless, socially aggressive behavior. d. socially anxious, rambling stories, peculiar ideas.

ANS: D Individuals with schizotypal personality disorder do not want to be involved in relationships. They are shy and introverted, speak little, and prefer fantasy and daydreaming to being involved with real people. The other behaviors would characteristically be noted in narcissistic, histrionic, and antisocial personality disorder. (The educator may reformat this question as multiple response.)

12. Which medication to maintain abstinence would most likely be prescribed for patients with an addiction to either alcohol or opioids? a. Bromocriptine (Parlodel) b. Methadone (Dolophine) c. Disulfiram (Antabuse) d. Naltrexone (ReVia)

ANS: D Naltrexone (ReVia) is useful for treating both opioid and alcohol addiction. An opioid antagonist blocks the action of opioids and the mechanism of reinforcement. It also reduces or eliminates alcohol craving.

20. Family members of an individual undergoing a residential alcohol rehabilitation program ask, "How can we help?" Select the nurse's best response. a. "Alcoholism is a lifelong disease. Relapses are expected." b. "Use search and destroy tactics to keep the home alcohol free." c. "It's important that you visit your family member on a regular basis." d. "Make your loved one responsible for the consequences of behavior."

ANS: D Often, the addicted individual has been enabled when others picked up the pieces for him or her. The individual never faced the consequences of his or her own behaviors, all of which relate to taking responsibility. Learning to face those consequences is part of the recovery process. The other options are codependent behaviors or are of no help.

1. A health care provider recently convicted of Medicare fraud says to a nurse, "Sure I overbilled. Everyone takes advantage of the government. There are too many rules to follow and I should get the money." These statements show: a. shame. b. suspiciousness c. superficial remorse. d. lack of guilt feelings.

ANS: D Rationalization is being used to explain behavior and deny wrongdoing. The individual who does not believe he or she has done anything wrong will not manifest anxiety, remorse, or guilt about the act. The patient's remarks cannot be assessed as shameful. Lack of trust and concern that others are determined to do harm is not shown.

22. A patient with an antisocial personality disorder was treated several times for substance abuse, but each time the patient relapsed. Which treatment approach is most appropriate? a. 1-week detoxification program b. Long-term outpatient therapy c. 12-step self-help program d. Residential program

ANS: D Residential programs and therapeutic communities help patients change lifestyles, abstain from drugs, eliminate criminal behaviors, develop employment skills, be self-reliant, and practice honesty. Residential programs are more effective for patients with antisocial tendencies than outpatient programs.

17. When preparing to interview a patient diagnosed with narcissistic personality disorder, a nurse can anticipate the assessment findings will include: a. preoccupation with minute details; perfectionist. b. charm, drama, seductiveness; seeking admiration. c. difficulty being alone; indecisive, submissiveness. d. grandiosity, self-importance, and a sense of entitlement

ANS: D The characteristics of grandiosity, self-importance, and entitlement are consistent with narcissistic personality disorder. Charm, drama, seductiveness, and admiration seeking are seen in patients with histrionic personality disorder. Preoccupation with minute details and perfectionism are seen in individuals with obsessive-compulsive personality disorder. Patients with dependent personality disorder often express difficulty being alone and are indecisive and submissive.

8. A patient diagnosed with an alcohol abuse disorder says, "Drinking helps me cope with being a single parent." Which therapeutic response by the nurse would help the patient conceptualize the drinking objectively? a. "Sooner or later, alcohol will kill you. Then what will happen to your children?" b. "I hear a lot of defensiveness in your voice. Do you really believe this?" c. "If you were coping so well, why were you hospitalized again?" d. "Tell me what happened the last time you drank."

ANS: D The correct response will help the patient see alcohol as a cause of the problems, not a solution, and begin to take responsibility. This approach can help the patient become receptive to the possibility of change. The other responses directly confront and attack defenses against anxiety that the patient still needs. They reflect the nurse's frustration with the patient.

21. Which goal for treatment of alcoholism should the nurse address first? a. Learn about addiction and recovery. b. Develop alternate coping strategies. c. Develop a peer support system. d. Achieve physiologic stability.

ANS: D The individual must have completed withdrawal and achieved physiologic stability before he or she is able to address any of the other treatment goals.

8. A patient's spouse filed charges after repeatedly being battered. The patient sarcastically says, "I'm sorry for what I did. I need psychiatric help." Which statement by the patient supports an antisocial personality disorder? a. "I have a quick temper, but I can usually keep it under control." b. "I've done some stupid things in my life, but I've learned a lesson." c. "I'm feeling terrible about the way my behavior has hurt my family." d. "I hit because I am tired of being nagged. My spouse deserves the beating."

ANS: D The patient with an antisocial personality disorder often impulsively acts out feelings of anger and feels no guilt or remorse. Patients with antisocial personality disorders rarely seem to learn from experience or feel true remorse. Problems with anger management and impulse control are common.

5. A patient admitted yesterday for injuries sustained while intoxicated believes bugs are crawling on the bed. The patient is anxious, agitated, and diaphoretic. What is the priority nursing diagnosis? a. Disturbed sensory perception b. Ineffective coping c. Ineffective denial d. Risk for injury

ANS: D The patient's clouded sensorium, sensory perceptual distortions, and poor judgment predispose a risk for injury. Safety is the nurse's priority. The other diagnoses may apply but are not the priorities of care.

29. A patient is thin, tense, jittery, and has dilated pupils. The patient says, "My heart is pounding in my chest. I need help." The patient allows vital signs to be taken but then becomes suspicious and says, "You could be trying to kill me." The patient refuses further examination. Abuse of which substance is most likely? a. PCP b. Heroin c. Barbiturates d. Amphetamines

ANS: D The physical symptoms are consistent with CNS stimulation. Suspicion and paranoid ideation are also present. Amphetamine use is likely. PCP use would probably result in bizarre, violent behavior. Barbiturates and heroin would result in symptoms of CNS depression.

18. For which behavior would limit setting be most essential? The patient who: a. clings to the nurse and asks for advice about inconsequential matters. b. is flirtatious and provocative with staff members of the opposite sex. c. is hypervigilant and refuses to attend unit activities. d. urges a suspicious patient to hit anyone who stares.

ANS: D This is a manipulative behavior. Because manipulation violates the rights of others, limit setting is absolutely necessary. Furthermore, limit setting is necessary in this case because the safety of at least two other patients is at risk. Limit setting may occasionally be used with dependent behavior (clinging to the nurse) and histrionic behavior (flirting with staff members), but other therapeutic techniques are also useful. Limit setting is not needed for a patient who is hypervigilant and refuses to attend unit activities; rather, the need to develop trust is central to patient compliance.

34. A patient took a large quantity of bath salts. Priority nursing and medical measures include: (select all that apply) a. administration of naloxone (Narcan). b. vitamin B12 and folate supplements. c. restoring nutritional integrity. d. management of heart rate. e. environmental safety.

ANS: D, E Care of patients who have taken bath salts is similar to those who have used other stimulants. Tachycardia and chest pain are common when a patient has used bath salts. These problems are life-threatening and take priority. Patients who have used these substances commonly have bizarre behavior and/or paranoia; therefore, safety is a priority concern. Nutrition is not a priority in an overdose situation. Vitamin replacements and naloxone apply to other drugs of abuse.

Client Needs: Psychosocial Integrity 17. A victim of a sexual assault comes to the hospital for treatment but abruptly decides to decline treatment and leaves the facility. While respecting the person's rights, the nurse should: a. say, "You may not leave until you receive prophylactic treatment for sexually transmitted diseases." b. provide written information about physical and emotional reactions the person may experience. c. explain the need and importance of infectious disease and pregnancy tests. d. give verbal information about legal resources in the community.

B All information given to a patient before he or she leaves the emergency department should be in writing. Patients who are anxious are unable to concentrate and therefore cannot retain much of what is verbally imparted. Written information can be read and referred to later. Patients may not be kept against their will or coerced into treatment. This constitutes false imprisonment. PTS: 1 DIF: Cognitive Level: Apply (Application) REF: Page 558 (Box 29-1) | Page 561 TOP: Nursing Process: Implementation

Client Needs: Psychosocial Integrity 18. An unconscious teenager is treated in the emergency department. The teenager's friends suspect a rape occurred at a party. Priority action by the nurse should focus on: a. preserving rape evidence. b. maintaining physiologic stability. c. determining what drugs were ingested. d. obtaining a description of the rape from a friend.

B Because the patient is unconscious, the risk for airway obstruction is present. The nurse's priority will focus on maintaining physiologic stability. The distracters are of lower priority than preserving physiological functioning. PTS: 1 DIF: Cognitive Level: Analyze (Analysis) REF: Page 556-558 TOP: Nursing Process: Planning

Client Needs: Psychosocial Integrity 11. A nurse interviews a 17-year-old male victim of sexual assault. The victim is reluctant to talk about the experience. Which comment should the nurse offer to this victim? a. "Male victims of sexual assault are usually better equipped than women to deal with the emotional pain that occurs." b. "Male victims of sexual assault often experience physical injuries and are assaulted by more than one person." c. "Do you have any male friends who have also been victims of sexual assault?" d. "Why do you think you became a victim of sexual assault?"

B Few rape survivors seek help, even with serious injury; so, it is important for the nurse to help the victim discuss the experience. The correct response therapeutically gives information to this victim. A male rape victim is more likely to experience physical trauma and to have been victimized by several assailants. Males experience the same devastation, physical injury, and emotional consequences as females. Although they may cover their responses, they too benefit from care and treatment. "Why" questions represent probing, which is a non-therapeutic communication technique. The victim may or may not have friends who have had this experience, but it's important to talk about his feelings rather than theirs. PTS: 1 DIF: Cognitive Level: Apply (Application) REF: Page 553-554 TOP: Nursing Process: Implementation

Client Needs: Physiological Integrity 14. Which situation describes consensual sex rather than rape? a. A husband forces vaginal sex when he comes home intoxicated from a party. The wife objects. b. A woman's lover pleads with her to have oral sex. She gives in but later regrets the decision. c. A person is beaten, robbed, and forcibly subjected to anal penetration by an assailant. d. A dentist gives anesthesia for a procedure and then has intercourse with the unconscious patient.

B Only the key describes a scenario in which the sexual contact is consensual. Consensual sex is not considered rape if the participants are of legal age. PTS: 1 DIF: Cognitive Level: Understand (Comprehension) REF: Page 553 TOP: Nursing Process: Assessment

Client Needs: Psychosocial Integrity 5. An emergency department nurse prepares to assist with evidence collection for a sexual assault victim. Prior to photographs and pelvic examination, what documentation is important? a. The patient's vital signs b. Consent signed by the patient c. Supervision and credentials of the examiner d. Storage location of the patient's personal effects

B Patients have the right to refuse legal and medical examination. Consent forms are required to proceed with these steps. PTS: 1 DIF: Cognitive Level: Understand (Comprehension) REF: Page 557-558 (Box 29-1) | Page 560 (Nursing Care Plan 29-1) TOP: Nursing Process: Implementation

MULTIPLE CHOICE 1. Which assessment data would help the health care team distinguish symptoms of conversion (functional neurological) disorder from symptoms of illness anxiety disorder (hypochondriasis)? a. Voluntary control of symptoms b. Patient's style of presentation c. Results of diagnostic testing d. The role of secondary gains

B Patients with illness anxiety disorder (hypochondriasis) tend to be more anxious about their concerns and display more obsessive attention to detail, whereas the patient with conversion (functional neurological) disorder often exhibits less concern with the symptom they are presenting than would be expected. Neither disorder involves voluntary control of the symptoms. Results of diagnostic testing for both would be negative (i.e., no physiological basis would be found for the symptoms). Secondary gains can occur in both disorders but are not necessary to either. See relationship to audience response question. PTS: 1 DIF: Cognitive Level: Apply (Application) REF: Page 325-327 TOP: Nursing Process: Assessment

Client Needs: Psychosocial Integrity 15. A patient reports fears of having cervical cancer and says to the nurse, "I've had Pap smears by six different doctors. The results were normal, but I'm sure that's because of errors in the laboratory." Which disorder would the nurse suspect? a. Conversion (functional neurological) disorder b. Illness anxiety disorder (hypochondriasis) c. Somatic symptom disorder d. Factitious disorder

B Patients with illness anxiety disorder have fears of serious medical problems, such as cancer or heart disease. These fears persist despite medical evaluations and interfere with daily functioning. There are no complaints of pain. There is no evidence of factitious or conversion disorder. PTS: 1 DIF: Cognitive Level: Understand (Comprehension) REF: Page 325-326 TOP: Nursing Process: Assessment

Which nursing diagnosis is the priority when planning care for a client who displays considerable anger and occasional aggression? A. Social isolation B. Risk for other-directed violence C. Ineffective coping: overwhelmed D. Ineffective coping: maladaptive

B. Risk for other-directed violence Risk for other-directed violence is the priority diagnosis. The nurse then must determine which of two other diagnoses—ineffective coping: overwhelmed or ineffective coping: maladaptive—is appropriate. Social isolation is not an initial concern.REF: 519

Client Needs: Psychosocial Integrity 3. A medical-surgical nurse works with a patient diagnosed with a somatic symptom disorder. Care planning is facilitated by understanding that the patient will probably: a. readily seek psychiatric counseling. b. be resistant to accepting psychiatric help. c. attend psychotherapy sessions without encouragement. d. be eager to discover the true reasons for physical symptoms.

B Patients with somatic symptom disorders go from one health care provider to another trying to establish a physical cause for their symptoms. When a psychological basis is suggested and a referral for counseling offered, these patients reject both. PTS: 1 DIF: Cognitive Level: Understand (Comprehension) REF: Page 325 TOP: Nursing Process: Planning

Client Needs: Psychosocial Integrity 4. A patient has blindness related to conversion (functional neurological) disorder but is unconcerned about this problem. Which understanding should guide the nurse's planning for this patient? a. The patient is suppressing accurate feelings regarding the problem. b. The patient's anxiety is relieved through the physical symptom. c. The patient's optic nerve transmission has been impaired. d. The patient will not disclose genuine fears.

B Psychoanalytical theory suggests conversion reduces anxiety through production of a physical symptom symbolically linked to an underlying conflict. Conversion, not suppression, is the operative defense mechanism in this disorder. While some MRI studies suggest that patients with conversion disorder have an abnormal pattern of cerebral activation, there is no actual alternation of nerve transmission. The other distracters oversimplify the dynamics, suggesting that only dependency needs are of concern, or suggest conscious motivation (conversion operates unconsciously). See relationship to audience response question. PTS: 1 DIF: Cognitive Level: Understand (Comprehension) REF: Page 325-326 TOP: Nursing Process: Planning

Client Needs: Psychosocial Integrity 18. A patient says, "I know I have a brain tumor despite the results of the MRI. The radiologist is wrong. People who have brain tumors vomit, and yesterday I vomited all day." Which response by the nurse fosters cognitive reframing? a. "You do not have a brain tumor. The more you talk about it, the more it reinforces your belief." b. "Let's see if there are any other possible explanations for your vomiting." c. "You seem so worried. Let's talk about how you're feeling." d. "We need to talk about something else."

B Questioning the evidence is a cognitive reframing technique. Identifying causes other than the feared disease can be helpful in changing distorted perceptions. Distraction by changing the subject will not be effective. PTS: 1 DIF: Cognitive Level: Apply (Application) REF: Page 331-334 TOP: Nursing Process: Implementation

Client Needs: Psychosocial Integrity 3. After an abduction and rape at gunpoint by an unknown assailant, which assessment finding best indicates that a patient is in the acute phase of the rape-trauma syndrome? a. Decreased motor activity c. Flashbacks and dreams b. Confusion and disbelief d. Fears and phobias

B Reactions of the acute phase of the rape-trauma syndrome are shock, emotional numbness, confusion, disbelief, restlessness, and agitated motor activity. Flashbacks, dreams, fears, and phobias are seen in the long-term reorganization phase of the rape-trauma syndrome. Decreased motor activity by itself is not indicative of any particular phase. PTS: 1 DIF: Cognitive Level: Understand (Comprehension) REF: Page 554 | Page 558 (Table 29-2) | Page 560 (Nursing Care Plan 29-1) TOP: Nursing Process: Assessment

Client Needs: Safe, Effective Care Environment 6. A nurse in the emergency department assesses an unresponsive victim of rape. The victim's friend reports, "That guy gave her salty water before he raped her." Which question is most important for the nurse to ask of the victim's friend? a. "Does the victim have any kidney disease?" b. "Has the victim consumed any alcohol?" c. "What time was she given salty water?" d. "Did you witness the rape?"

B Salty water is a slang/street name for GHB (γ-hydroxy-butyric acid), a Schedule III central nervous system depressant associated with rape. Use of alcohol would produce an increased risk for respiratory depression. GHB has a duration of 1-12 hours, but the duration is less important that the potential for respiratory depression. Seeking evidence is less important than the victim's physiologic stability. PTS: 1 DIF: Cognitive Level: Analyze (Analysis) REF: Page 554 (Table 29-1) TOP: Nursing Process: Implementation

Client Needs: Psychosocial Integrity 5. A patient has blindness related to conversion (functional neurological) disorder. To help the patient eat, the nurse should: a. establish a "buddy" system with other patients who can feed the patient at each meal. b. expect the patient to feed self after explaining arrangement of the food on the tray. c. direct the patient to locate items on the tray independently and feed self. d. address needs of other patients in the dining room, then feed this patient.

B The patient is expected to maintain some level of independence by feeding self, while the nurse is supportive in a matter-of-fact way. The distracters support dependency or offer little support. PTS: 1 DIF: Cognitive Level: Apply (Application) REF: Page 335-336 TOP: Nursing Process: Implementation

Client Needs: Psychosocial Integrity 6. A patient with blindness related to conversion (functional neurological) disorder says, "All the doctors and nurses in the hospital stop by often to check on me. Too bad people outside the hospital don't find me as interesting." Which nursing diagnosis is most relevant? a. Social isolation c. Interrupted family processes b. Chronic low self-esteem d. Ineffective health maintenance

B The patient mentions that the symptoms make people more interested. This indicates that the patient feels uninteresting and unpopular without the symptoms, thus supporting the nursing diagnosis of chronic low self-esteem. Defining characteristics for the other nursing diagnoses are not present in the scenario. PTS: 1 DIF: Cognitive Level: Apply (Application) REF: Page 325-326 TOP: Nursing Process: Diagnosis/Analysis

Client Needs: Psychosocial Integrity 11. A patient with a somatic symptom disorder has the nursing diagnosis Interrupted family processes related to patient's disabling symptoms as evidenced by spouse and children assuming roles and tasks that previously belonged to patient. An appropriate outcome is that the patient will: a. assume roles and functions of other family members. b. demonstrate performance of former roles and tasks. c. focus energy on problems occurring in the family. d. rely on family members to meet personal needs.

B The patient with a somatic symptom disorder has typically adopted a sick role in the family, characterized by dependence. Increasing independence and resumption of former roles are necessary to change this pattern. The distracters are inappropriate outcomes. PTS: 1 DIF: Cognitive Level: Apply (Application) REF: Page 325-326 | Page 338 (Nursing Care Plan 17-1) TOP: Nursing Process: Outcomes Identification

Client Needs: Psychosocial Integrity 2. Which prescription medication would the nurse expect to be prescribed for a patient diagnosed with a somatic symptom disorder? a. Narcotic analgesics for use as needed for acute pain b. Antidepressant medications to treat underlying depression c. Long-term use of benzodiazepines to support coping with anxiety d. Conventional antipsychotic medications to correct cognitive distortions

B Various types of antidepressants may be helpful in somatic disorders directly by reducing depressive symptoms and hence somatic responses, but also indirectly by affecting nerve circuits that affect not only mood, but fatigue, pain perception, GI distress, and other somatic symptoms. Patients may benefit from short-term use of anti-anxiety medication (benzodiazepines) but require careful monitoring because of risks of dependence. Conventional antipsychotic medications would not be used, although selected atypical antipsychotics may be useful. Narcotic analgesics are not indicated. PTS: 1 DIF: Cognitive Level: Understand (Comprehension) REF: Page 335-336 TOP: Nursing Process: Planning

Client Needs: Psychosocial Integrity 5. A rape victim tells the emergency nurse, "I feel so dirty. Help me take a shower before I get examined." The nurse should: (select all that apply) a. arrange for the victim to shower. b. explain that bathing destroys evidence. c. give the victim a basin of water and towels. d. offer the victim a shower after evidence is collected. e. explain that bathing facilities are not available in the emergency department.

B, D As uncomfortable as the victim may be, she should not bathe until the examination is completed. Collection of evidence is critical for prosecution of the attacker. Showering after the examination will provide comfort to the victim. The distracters will result in destruction of evidence or are untrue. PTS: 1 DIF: Cognitive Level: Apply (Application) REF: Page 557-558 (Box 29-1) TOP: Nursing Process: Implementation

Client Needs: Psychosocial Integrity MULTIPLE RESPONSE 1. A child has a history of multiple hospitalizations for recurrent systemic infections. The child is not improving in the hospital, despite aggressive treatment. Factitious disorder by proxy is suspected. Which nursing interventions are appropriate? Select all that apply. a. Increase private visiting time for the parents to improve bonding. b. Keep careful, detailed records of visitation and untoward events. c. Place mittens on the child to reduce access to ports and incisions. d. Encourage family members to visit in groups of two or three. e. Interact with the patient frequently during visiting hours.

B, D, E Factitious disorder by proxy is a rare condition wherein a person intentionally causes or perpetuates the illness of a loved one (e.g., by periodically contaminating IV solutions with fecal material). When this disorder is suspected, the child's life could be at risk. Depending on the evidence supporting this suspicion, interventions could range from minimizing unsupervised visitation to blocking visitation altogether. Frequently checking on the child during visitation and minimizing unobserved access to the child (by encouraging small group visits) reduces the opportunity to take harmful action and increases the collection of data that can help determine whether this disorder is at the root of the child's illness. Detailed tracking of visitation and untoward events helps identify any patterns there might be between select visitors and the course of the child's illness. Increasing private visitation provides more opportunity for harm. Educating visitors about aseptic techniques would not be of help if the infections are intentional, and preventing inadvertent contamination by the child himself would not affect factitious disorder by proxy. PTS: 1 DIF: Cognitive Level: Apply (Application) REF: Page 337-338 TOP: Nursing Process: Implementation

Ian makes the following statements to you while admitting him. Which statement indicates and increased likelihood of violent behavior? A. "When I get mad, I want to be left alone" B. "Last time I was in here I ended up in seclusion for punching my roommate C. "My old man was meek and mild, and I've always said I'm not going to be like him" D. "My girlfriend says I yell way too much, and she's threatened to leave me."

B. "Last time I was in here I ended up in seclusion for punching my roommate

Conversion disorder is described as an absence of a neurological diagnosis that manifests in neurological symptoms. Channeling of emotions, conflicts, and stressors into physical symptoms is thought to be the cause in conversion disorder. Which statement is true? A. People with conversion disorder are extremely upset about often dramatic symptoms. B. Abnormal patterns of cerebral activation have been found in individuals with conversion disorder. C. An organic cause is usually found in most cases of conversion disorder. D. Symptoms can be turned on and off depending on the patient's choice

B. Abnormal patterns of cerebral activation have been found in individuals with conversion disorder.

A nurse is assessing a client in an inpatient mental health unit. Which of the following findings should the nurse expect if the client is in the preassaultive stage of violence? (select all that apply) A. Lethargy B. Defensive responses to questions C. Disorientation D. Facial grimacing E. Agitation

B. Defensive responses to questions D. Facial grimacing E. Agitation

When you approach Katy, what considerations should you take? A. Stand close to Katy for reassurance and to convey caring. B. Have other staff as backup, and stand far enough away to avoid injury. C. Take Katy to her room so you can speak with her alone. D. Call security and wait until they arrive before approaching Katy.

B. Have other staff as backup, and stand far enough away to avoid injury. Safety considerations for staff include enlisting other staff to be present, keeping a safe distance from the patient, and approaching the patient in a nonthreatening or nonconfrontational manner. The other options do not allow for staff safety; security personnel may escalate the patient's behavior and should be kept in the background until needed to assist. Furthermore, the patient has an immediate need to be assisted by staff if possible without waiting for security.Cognitive Level: Analyze (Analysis)Nursing Process: PlanningNCLEX: Safe and Effective Care EnvironmentText page: 520

An angry client frequently loses patience with the nurses and shouts at them while they perform a complicated dressing change. Which plan could they create to intervene effectively in this behavior? A. Tell him they will not change his dressing if he is going to abuse them. B. When the client begins to become abusive, leave the room promising to return in 20 minutes when he has regained control. C. Assure him they will complete the dressing change as quickly as possible. D. Explain that they are professionals and unused to being shouted at by people they are trying to help.

B. When the client begins to become abusive, leave the room promising to return in 20 minutes when he has regained control. The nurse is using behavioral techniques to reinforce desirable behavior (spending time with the client when he is calm) and limit reinforcement of undesirable behavior (leaving when he is acting out anger).REF: Page 521, 525

When a client diagnosed with a cognitive deficit experiences a catastrophic reaction, the priority intervention is to A. decrease sensory stimuli. B. smile and call the client by name. C. take the client to the bathroom. D. calmly ask the client what's wrong. b

B. smile and call the client by name. Getting the client's attention by calling his or her name is necessary. Smiling is necessary to convey the lack of a threat.REF: Page 527

One older concept that is being used currently that may help in violence reduction in patients is: A. aired grievances. B. trauma-informed care. C. shared governance. D. learned helplessness.

B. trauma-informed care.' Trauma-informed care is an older concept of providing care that has been reintroduced. It is based on the notion that disruptive patients often have histories that include violence and victimization. These traumatic histories can impede patients' ability to self-soothe, result in negative coping responses, and create a vulnerability to coercive interventions (e.g., restraint) by staff. Trauma-informed care focuses on the patients' past experiences of violence or trauma and the role it currently plays in their lives. The other options do not refer to a care concept that helps reduce violence.Cognitive Level: Understand (Comprehension)Nursing Process: PlanningNCLEX: Psychosocial IntegrityText page: 517

Ever since participating in a village raid where explosives were used, a military veteran has been unable to walk. After all diagnostic testing were negative for any physical abnormalities, the client was diagnosed with conversion disorder. What is the nurse's best response when asked by the client, "Why can't I walk?" A. "Your legs don't work because your brain is screwed up." B."Your emotional distress is being expressed as a physical symptom." C. "You are making up your symptoms as a cry for help." D. "You are overly anxious about having a severe illness."

B."Your emotional distress is being expressed as a physical symptom." Conversion disorder is attributed to channeling of emotional conflicts or stressors into physical symptoms. Telling the patient her brain is "screwed up" is unprofessional and does not give any useful education. Symptoms of conversion disorder are not within the patient's voluntary control. Being overly anxious about having a severe illness describes illness anxiety disorder

Client Needs: Psychosocial Integrity 21. A patient diagnosed with a somatic symptom disorder has been in treatment for 4 weeks. The patient says, "Although I'm still having pain, I notice it less and am able to perform more activities." The nurse should evaluate the treatment plan as: a. marginally successful. c. partially successful. b. minimally successful. d. totally achieved.

C Decreased preoccupation with symptoms and increased ability to perform activities of daily living suggest partial success of the treatment plan. Total success is rare because of patient resistance. PTS: 1 DIF: Cognitive Level: Apply (Application) REF: Page 333 | Page 337 TOP: Nursing Process: Evaluation

Client Needs: Psychosocial Integrity 10. A rape victim visited a rape crisis counselor weekly for 8 weeks. At the end of this counseling period, which comment by the victim best demonstrates that reorganization was successful? a. "I have a rash on my buttocks. It itches all the time." b. "Now I know what I did that triggered the attack on me." c. "I'm sleeping better although I still have an occasional nightmare." d. "I have lost 8 pounds since the attack, but I needed to lose some weight."

C Rape-trauma syndrome is a variant of posttraumatic stress disorder. The absence of signs and symptoms of posttraumatic stress disorder suggest that the long-term reorganization phase was successfully completed. The victim's sleep has stabilized; occasional nightmares occur, even in reorganization. The distracters suggest somatic symptoms, appetite disturbances, and self-blame, all of which are indicators that the process is ongoing. PTS: 1 DIF: Cognitive Level: Apply (Application) REF: Page 562 TOP: Nursing Process: Evaluation

Client Needs: Psychosocial Integrity 16. A patient diagnosed with a somatic symptom disorder says, "My pain is from an undiagnosed injury. I can't take care of myself. I need pain medicine six or seven times a day. I feel like a baby because my family has to help me so much." It is important for the nurse to assess: a. mood. c. secondary gains. b. cognitive style. d. identity and memory.

C Secondary gains should be assessed. The patient's dependency needs may be met through care from the family. When secondary gains are prominent, the patient is more resistant to giving up the symptom. The scenario does not allude to a problem of mood. Cognitive style and identity and memory assessment are of lesser concern because the patient's diagnosis has been established. PTS: 1 DIF: Cognitive Level: Apply (Application) REF: Page 331-333 (Table 17-3) | Page 335 (Table 17-4) TOP: Nursing Process: Assessment

Client Needs: Psychosocial Integrity 7. To assist patients diagnosed with somatic symptom disorders, nursing interventions of high priority: a. explain the pathophysiology of symptoms. b. help these patients suppress feelings of anger. c. shift focus from somatic symptoms to feelings. d. investigate each physical symptom as it is reported.

C Shifting the focus from somatic symptoms to feelings or to neutral topics conveys interest in the patient as a person rather than as a condition. The need to gain attention with the use of symptoms is reduced over the long term. A desired outcome would be that the patient would express feelings, including anger if it is present. Once physical symptoms are investigated, they do not need to be reinvestigated each time the patient reports them. PTS: 1 DIF: Cognitive Level: Understand (Comprehension) REF: Page 333-335 (Table 17-4) TOP: Nursing Process: Implementation

Client Needs: Psychosocial Integrity 19. Which treatment modality should a nurse recommend to help a patient diagnosed with a somatic symptom disorder to cope more effectively? a. Flooding c. Relaxation techniques b. Response prevention d. Systematic desensitization

C Somatic symptom disorders are commonly associated with complicated reactions to stress. These reactions are accompanied by muscle tension and pain. Relaxation can diminish the patient's perceptions of pain and reduce muscle tension. The distracters are modalities useful in treating selected anxiety disorders. PTS: 1 DIF: Cognitive Level: Apply (Application) REF: Page 334-335 TOP: Nursing Process: Planning

You are caring for Malcolm, an 83 y/o African American patient with Alzheimer's disease. Malcolm exhibits agitated behavior at times, especially when he feels he is missing work, and he sometimes attempts to leave the unit to " get to the school where I teach. " which of the following interventions is appropriate for de-escalating Malcolm's agitation. A. Medicate Malcolm with PRN medication at regular intervals to prevent agitation B. Repeatedly explain to Malcolm that he is retired and no longer teaches as the repetition will reinforce the patient's orientation C . Use validation therapy and ask Malcolm about the school and his job D. Reduce stimulation In the environment by having Malcolm sit by himself in his room until the agitation passes

C . Use validation therapy and ask Malcolm about the school and his job

Which experiences are most likely to precipitate posttraumatic stress disorder (PTSD)? Select all that apply. a. A young adult bungee jumped from a bridge with a best friend. b. An 8-year-old child watched an R-rated movie with both parents. c. An adolescent was kidnapped and held for 2 years in the home of a sexual predator. d. A passenger was in a bus that overturned on a sharp curve and tumbled down an embankment. e. An adult was trapped for 3 hours at an angle in an elevator after a portion of the supporting cable breaks.

C, D, E PTSD usually occurs after a traumatic event that is outside the range of usual experience. Examples are childhood physical abuse, torture/kidnap, military combat, sexual assault, and natural disasters, such as floods, tornados, earthquakes, tsunamis; human disasters, such as a bus or elevator accident; or crime-related events, such being taken hostage. The common element in these experiences is the individuals extraordinary helplessness or powerlessness in the face of such stressors. Bungee jumps by adolescents are part of the developmental task and might be frightening, but in an exhilarating way rather than a harmful way. A child may be disturbed by an R-rated movie, but the presence of the parents would modify the experience in a positive way.

Diane, a 63 year-old mother of three, was brought to the community psychiatric clinic. Diane and her son had a bitter fight over finances. Ever since Diane has been complaining of a "severe pain in my neck." She has seen several doctors who cannot find a physical basis for the pain. The nurse knows that: A. Showing for concern for Diane's pain will increase her obsessional thinking. B. Diane's symptoms are manipulative and under conscious control. C. Diane believes there is a physical cause for the pain and will resist a psychological explanation. D. Diane is trying to make her son feel bad about the argument.

C. Diane believes there is a physical cause for the pain and will resist a psychological explanation.

Which statementt about violence and nursing is accurate? A. Unless working in psychiatric mental health settings, nurses are unlikely to experience patient violence B. To date, no legislation exists that addresses workplace violence against nurses C. Emergency, psychiatric, and step-down units have the highest rates of violence towards staff D. Violence primarily affects inexperienced or unskilled staff who cannot calm their patients

C. Emergency, psychiatric, and step-down units have the highest rates of violence towards staff

A nurse is caring for a client in an inpatient mental health facility who gets up from a chair and throws it across the day room. Which of the following is the priority nursing action? A. Encourage the client to express her feelings B. Maintain eye contact with the client C. Move the client away from others D. Tell the client that the behavior is not acceptable

C. Move the client away from others

The factor most likely to contribute to a client's escalating anger is A. watching violence on television. B. another client's depressed mood. C. a staff member telling him that he is inappropriate. D. a staff member asking him to help another client.

C. a staff member telling him that he is inappropriate. Punitive, threatening, accusatory, or challenging statements to the client should be avoided; rather, the nurse should determine what is behind the client's feelings and behaviors.REF: 520

The client at highest risk for violence directed at others is one who A. has a history of recurrent severe depression. B. is in an alcohol rehabilitation program. C. has delusions of persecution. D. who has somatic symptoms for which no organic basis is found.

C. has delusions of persecution. The client who perceives others to be against him may lash out if he feels threatened.REF: Page 518

When working with an angry client, it is best to A. encourage the client to fully explore and express his or her anger. B. help the client deny and repress the feelings of anger C. help the client reframe the anger-producing situation. D. ignore the client's anger and change the subject.

C. help the client reframe the anger-producing situation. De-escalation occurs more quickly with this strategy than when other approaches are used.REF: 518-519

An adolescent male is swearing and shouting at his physician, who refused to give him a pass to leave the unit. This behavior A. is acceptable if directed at staff but not when directed at other clients. B. may reduce tension and prevent the client from physically acting out. C. is a major indicator that the client may become physically aggressive. D. can be attributed to lack of parental controls applied at an early age.

C. is a major indicator that the client may become physically aggressive. Physical aggression is preceded by anger, which may be expressed by swearing and shouting, pacing, and other menacing behaviors.REF: Page 520

A nurse attempts to intervene verbally when an angry client initially threatens to throw a chair but quickly focuses the anger toward the nurse. Several staff members gather behind the nurse, but then the client shouts, "I will calm down when that nurse isn't in my face." The nurse best demonstrates the ability to help the client deescalate by A. continuing to manage the situation personally. B. telling the client, "It isn't safe for me to leave the room." C. moving to the rear of the staff group. D. apologizing for upsetting the client.

C. moving to the rear of the staff group. There is no need for the nurse to stand her ground to save face. The goal is to deescalate the situation. When the client makes a request that can be met without compromising safety, granting the request is acceptable.REF: Page 520-521

Peter, a 21-year-old patient, asks you, "What's wrong with my brain that I have such a problem with aggression?" Your response is based on the knowledge that: A. the prevailing theory is that diminishment of stress hormones causes anger and aggression. B. no abnormalities of the brain have been identified that correlate with anger and aggression. C. the limbic system, the prefrontal cortex, and neurotransmitters have been implicated in playing a part in aggression. D. personality type plays a much greater part in anger and aggression than physical factors.

C. the limbic system, the prefrontal cortex, and neurotransmitters have been implicated in playing a part in aggression. These have all been implicated by research as playing a part in anger and aggression. The other responses are untrue.Cognitive Level: Apply (Application)Nursing Process: ImplementationNCLEX: Psychosocial IntegrityText page: 516

What would be an appropriate expected outcome of the treatment plan for a client diagnosed with a conversion disorder that interferes with the ability to walk effective? A. Client will walk unassisted within 1 week. B. Client will return to a pre-illness level of functioning within 2 weeks. C. Client will be able to state two new effective coping skills within 2 weeks. D. Client will assume full self-care within 3 weeks.

Client will be able to state two new effective coping skills within 2 weeks. An appropriate outcome for somatization disorders is to be aware of negative coping strategies and learn new, effective skills for coping within a realistic timeframe. In the other options, the time frames of these outcomes are unrealistic

Client Needs: Psychosocial Integrity 8. A rape victim tells the nurse, "I should not have been out on the street alone." Select the nurse's most therapeutic response. a. "Rape can happen anywhere." b. "Blaming yourself increases your anxiety and discomfort." c. "You are right. You should not have been alone on the street at night." d. "You feel as though this would not have happened if you had not been alone."

D A reflective communication technique is most helpful. Looking at one's role in the event serves to explain events that the victim would otherwise find incomprehensible. The distracters discount the victim's perceived role and interfere with further discussion. PTS: 1 DIF: Cognitive Level: Apply (Application) REF: Page 556-557,559-560 TOP: Nursing Process: Implementation

Client Needs: Psychosocial Integrity 2. A woman was found confused and disoriented after being abducted and raped at gunpoint by an unknown assailant. The emergency department nurse makes these observations about the woman: talking rapidly in disjointed phrases, unable to concentrate, indecisive when asked to make simple decisions. What is the woman's level of anxiety? a. Weak c. Moderate b. Mild d. Severe

D Acute anxiety results from the personal threat to the victim's safety and security. In this case, the patient's symptoms of rapid, dissociated speech, inability to concentrate, and indecisiveness indicate severe anxiety. Weak is not a level of anxiety. Mild and moderate levels of anxiety would allow the patient to function at a higher level. PTS: 1 DIF: Cognitive Level: Understand (Comprehension) REF: Page 560 (Nursing Care Plan 29-1) + learning from Chapter 15 TOP: Nursing Process: Assessment

Client Needs: Psychosocial Integrity 9. The nursing diagnosis Rape-trauma syndrome applies to a rape victim in the emergency department. Select the most appropriate outcome to achieve before discharging the patient. a. The memory of the rape will be less vivid and less frightening. b. The patient is able to describe feelings of safety and relaxation. c. Symptoms of pain, discomfort, and anxiety are no longer present. d. The patient agrees to a follow-up appointment with a rape victim advocate.

D Agreeing to keep a follow-up appointment is a realistic short-term outcome. The victim is in the acute phase; the distracters are unlikely to be achieved during the limited time the victim is in an emergency department. PTS: 1 DIF: Cognitive Level: Apply (Application) REF: Page 556 | Page 558 | Page 561 TOP: Nursing Process: Planning/Outcomes Identification

Client Needs: Psychosocial Integrity 10. To plan effective care for patients diagnosed with somatic symptom disorders, the nurse should understand that patients have difficulty giving up the symptoms because the symptoms: a. are generally chronic. c. can be voluntarily controlled. b. have a physiological basis. d. provide relief from health anxiety.

D At the unconscious level, the patient's primary gain from the symptoms is anxiety relief. Considering that the symptoms actually make the patient more psychologically comfortable and may also provide secondary gain, patients frequently fiercely cling to the symptoms. The symptoms tend to be chronic, but that does not explain why they are difficult to give up. The symptoms are not under voluntary control or physiologically based. PTS: 1 DIF: Cognitive Level: Understand (Comprehension) REF: Page 325-326 | Page 338 (Nursing Care Plan 17-1) TOP: Nursing Process: Planning

Chapter 29: Sexual Assault MULTIPLE CHOICE 1. The nurse at a university health center leads a dialogue with female freshmen about rape and sexual assault. One student says, "If I avoid strangers or situations where I am alone outside at night, I'll be safe from sexual attacks." Choose the nurse's best response. a. "Your plan is not adequate. You could still be raped or sexually assaulted." b. "I am glad you have this excellent safety plan. Would others like to comment?" c. "It's better to walk with someone or call security when you enter or leave a building." d. "Sexual assaults are more often perpetrated by acquaintances. Let's discuss ways to prevent that."

D Females know their offenders in almost 70% of all violent crimes committed against them, including rape. The nurse should share this information along with encouraging discussion of safety measures. The distracters fail to provide adequate information or encourage discussion. PTS: 1 DIF: Cognitive Level: Apply (Application) REF: Page 553-554 | Page 560 (Nursing Care Plan 29-1) TOP: Nursing Process: Assessment

Client Needs: Psychosocial Integrity 8. A patient with fears of serious heart disease was referred to the mental health center by a cardiologist. Extensive diagnostic evaluation showed no physical illness. The patient says, "My chest is tight, and my heart misses beats. I'm often absent from work. I don't go out much because I need to rest." Which health problem is most likely? a. Dysthymic disorder b. Somatic symptom disorder c. Antisocial personality disorder d. Illness anxiety disorder (hypochondriasis)

D Illness anxiety disorder (hypochondriasis) involves preoccupation with fears of having a serious disease even when evidence to the contrary is available. The preoccupation causes impairment in social or occupational functioning. Somatic symptom disorder involves fewer symptoms. Dysthymic disorder is a disorder of lowered mood. Antisocial disorder applies to a personality disorder in which the individual has little regard for the rights of others. See relationship to audience response question. PTS: 1 DIF: Cognitive Level: Apply (Application) REF: Page 325-326 TOP: Nursing Process: Assessment

Client Needs: Psychosocial Integrity 15. Before a victim of sexual assault is discharged from the emergency department, the nurse should: a. notify the victim's family to provide emotional support. b. offer to stay with the patient until stability is regained. c. advise the patient to try not to think about the assault. d. provide referral information verbally and in writing.

D Immediately after the assault, rape victims are often disorganized and unable to think well or remember instructions. Written information acknowledges this fact and provides a solution. The distracters violate the patient's right to privacy, evidence a rescue fantasy, and offer a platitude that is neither therapeutic nor effective. PTS: 1 DIF: Cognitive Level: Apply (Application) REF: Page 556 | Page 558 (Box 29-1) | Page 561 TOP: Nursing Process: Implementation

Client Needs: Safe, Effective Care Environment 13. A nurse cares for a rape victim who was given a drink that contained flunitrazepam (Rohypnol) by an assailant. Which intervention has priority? Monitoring for: a. coma. c. hypotonia. b. seizures. d. respiratory depression.

D Monitoring for respiratory depression takes priority over hypotonia, seizures, or coma. PTS: 1 DIF: Cognitive Level: Apply (Application) REF: Page 554 (Table 29-1) TOP: Nursing Process: Intervention

Client Needs: Psychosocial Integrity 17. What is an essential difference between somatic symptom disorders and factitious disorders? a. Somatic symptom disorders are under voluntary control, whereas factitious disorders are unconscious and automatic. b. Factitious disorders are precipitated by psychological factors, whereas somatic symptom disorders are related to stress. c. Factitious disorders are individually determined and related to childhood sexual abuse, whereas somatic symptom disorders are culture bound. d. Factitious disorders are under voluntary control, whereas somatic symptom disorders involve expression of psychological stress through somatization.

D The key is the only fully accurate statement. Somatic symptom disorders involve expression of stress through bodily symptoms and are not under voluntary control or culture bound. Factitious disorders are under voluntary control. See relationship to audience response question. PTS: 1 DIF: Cognitive Level: Understand (Comprehension) REF: Page 325-326 | Page 337-338 TOP: Nursing Process: Assessment

Client Needs: Psychosocial Integrity 9. A nurse assessing a patient diagnosed with a somatic symptom disorder is most likely to note that the patient: a. sees a relationship between symptoms and interpersonal conflicts. b. has little difficulty communicating emotional needs to others. c. rarely derives personal benefit from the symptoms. d. has altered comfort and activity needs.

D The patient frequently has altered comfort and activity needs associated with the symptoms displayed (fatigue, insomnia, weakness, tension, pain, etc.). In addition, hygiene, safety, and security needs may also be compromised. The patient is rarely able to see a relation between symptoms and events in his or her life, which is readily discernible to health professionals. Patients with somatic symptom disorders often derive secondary gain from their symptoms and/or have considerable difficulty identifying feelings and conveying emotional needs to others. PTS: 1 DIF: Cognitive Level: Understand (Comprehension) REF: Page 325-326 | Page 338 (Nursing Care Plan 17-1) TOP: Nursing Process: Assessment

Client Needs: Psychosocial Integrity 4. A nurse interviews a patient abducted and raped at gunpoint by an unknown assailant. The patient says, "I can't talk about it. Nothing happened. I have to forget." What is the patient's present coping strategy? a. Compensation c. Projection b. Somatization d. Denial

D The patient's statements reflect use of denial, an ego defense mechanism. This mechanism may be used unconsciously to protect the person from the emotionally overwhelming reality of the rape. The patient's statements do not reflect somatization, compensation, or projection. PTS: 1 DIF: Cognitive Level: Understand (Comprehension) REF: Page 558 (Table 29 - 2) + learning from Chapter 15 TOP: Nursing Process: Assessment

Emily asks you what kind of therapy will help her. Based on current knowledge, what form of therapy is most appropriate for a client diagnosed with a conversion disorder? A. "A combination of antianxiety and antidepressant therapy is the most effective therapy." B. "Aversion therapy is often used because in effect you are punishing yourself by not being able to walk." C. "Modeling will be used; as you see desired behaviors modeled by the therapist you will be able to also achieve the expected outcome." D. "Cognitive-behavioral therapy (CBT) has been shown to consistently provide the best outcome for these types of disorders."

D. CBT is the most consistently supported treatment for the full spectrum of somatic disorders. All the other options are incorrect and do not describe the most used and effective therapy for this disorder.

You are working in the emergency department. You notice Matt, your patient's husband, pacing in the hallway, muttering to himself, and looking angrily around the emergency department. Which of the following statements to Matt may help prevent escalation and/or violence? A. "You need to stay with your wife. She needs you." B. "Hey, what's up buddy? You look pissed." C."I am calling security to deal with your behavior." D. You appear upset. Can I help you with anything?"

D. "You appear upset. Can I help you with anything?" Approaching a patient or a visitor with a calm, sincere, and caring manner can de-escalate a situation because the person may feel you are interested in helping. The other responses will not prevent escalation and may in fact anger the person further.Cognitive Level: Analyze (Analysis)Nursing Process: ImplementationNCLEX: Safe and Effective Care EnvironmentText page: 520

A nurse is conducting group therapy with a group of clients. Which of the following statements made by a client is an example of aggressive communication? A. "I wish you would not make me angry" B. "I feel angry when you leave me." C. "It makes me angry when you interrupt me." D. "You'd better listen to me"

D. "You'd better listen to me"

Which patient is at greatest risk for developing a stress induced myocardial infarction? A. A patient who lost a child in an accidental shooting 24 hours ago B. A woman who has begun experiencing early signs of menopause C. A patient who has spent years trying to sustain a successful business D. A patient who was diagnosed with chronic depression 10 years ago

D. A patient who was diagnosed with chronic depression 10 years ago

You are caring for Aaron, a 38 year-old patient diagnosed with somatic disorder. When interacting with you, Aaron continues to focus on his severe headaches. In planning care for Aaron, which of the following interventions would be appropriate? A. Call for a family meeting with Aaron in attendance to confront Aaron regarding his diagnosis B. Educate Aaron on alternative therapies to deal with pain. C. Improve reality testing by telling Aaron that you do not believe the headaches are real D. After a limited discussion of physical concerns, shift focus to feelings and effective coping skills.

D. After a limited discussion of physical concerns, shift focus to feelings and effective coping skills

It is most important for the nurse to employ which holistic strategy when managing clients diagnosed with a somatization disorder? A. Utilizing many different therapeutic strategies or modalities for enhanced coping B. Involving every member of the family as well as the patient in treatment C. Incorporating spirituality and religion into treatment D. Considering all dimensions of the patient, including biological, psychological, and sociocultural

D. Considering all dimensions of the patient, including biological, psychological, and sociocultural It is important to use a holistic approach in nursing care so that we may address the multidimensional interplay of biological, psychological, and sociocultural needs and its effects on the somatization process. All nurses need to be aware of the influence of environment, stress, individual lifestyle, and coping skills of each patient. The other options do not explain the concept of holistic care.

Living comfortable and materialistic lives in Western societies seems to have altered the original hierarchy proposed by Maslow in that: A. Once lower level needs are satisfied, no further growth feels necessary B. Self-actualization is easier to achieve with financial stability C. Esteem is more highly valued than safety. D. Focusing on materialism reduces interests in love, belonging, and family

D. Focusing on materialism reduces interests in love, belonging, and family

Anger can best be defined as A. an unhealthy way of releasing anxiety. B. doing intentional harm to others. C. an expression of conflict with others. D. a normal response to a perceived threat.

D. a normal response to a perceived threat.

A client waiting to see the physician is pacing and looking both angry and tense. When it's determined that the client won't be seen for another 30 minutes, the nurse addresses the client's agitation by A. telling the client that pacing will not help the rate at which clients are seen. B. adjusting the appointment schedule to allow the client to be seen next. C. empathizing with the long wait and asking the client if he would mind sitting down until his turn comes. D. explaining to the client what caused the back-up and suggesting that he has time to go to the coffee shop.

D. explaining to the client what caused the back-up and suggesting that he has time to go to the coffee shop. Taking time to explain to clients and offering measures that will provide comfort can be helpful in reducing tension and anger associated with waiting.REF: Page 521

A client experiencing manic hyperactivity stands up, glares challengingly at clients and staff, and shouts, "This food is garbage! I'll fight anyone who says it's not!" The nurse's most relevant assessment is that the client A. is upset with the quality of the food. B. is getting rid of tension in a harmless way. C. is frustrated by limits imposed by hospitalization. D.has a high potential for other-directed violence.

D. has a high potential for other-directed violence. The client's offers to fight are suggestive of a high potential for violence. Clients may have coping skills that are adequate for day-to-day events in their lives but are overwhelmed by the stresses of illness or hospitalization. Other clients may have a pattern of maladaptive coping, which is marginally effective and consists of a set of coping strategies that have been developed to meet unusual or extraordinary situations.REF: Page 516-517

A client has a history of demonstrating aggression physically. An appropriate short-term goal to help the client manage this anger is to A. strike objects rather than people. B. limit aggression to verbal outbursts. C. isolate in lieu of striking people. D. identify situations that precipitate hostility.

D. identify situations that precipitate hostility. The identification of situations that create hostile feelings must occur if the client is to develop new coping strategies.REF: Page 527

A client has been placed in seclusion to control aggressive behavior. Care while the client is secluded should include A. observation every 30 minutes. B. releasing the client every 8 hours. C. releasing the client every 8 hours. D. providing for nutrition and hydration.

D. providing for nutrition and hydration. Clients must be given meals on schedule and frequently offered cold liquids in paper cups (at least every 2 hours; hourly if the client is highly hyperactive).REF: Page 521

Nurses coping with angry clients may find it helpful to remember that anger and aggression begin as feelings of A. isolation. B. confidence. C. competence. D. vulnerability.

D. vulnerability. The progression is vulnerability, perception of event as a threat, arousal, and then uneasiness and anxiety.REF: 526

1. Which statement made by the patient demonstrates an understanding of the treatment of choice for patients managing the effects of traumatic events? a. "I attend my therapy sessions regularly." b. "Those intrusive memories are hidden for a reason and should stay hidden." c. "Keeping busy is the key to getting mentally healthy." d. "I've agreed to move in with my parents so I'll get the support I need."

a. "I attend my therapy sessions regularly."

Which assessment finding best supports dissociative fugue? The patient states: a. "I cannot recall why I'm living in this town." b. "I feel as if I'm living in a fuzzy dream state." c. "I feel like different parts of my body are at war." d. "I feel very anxious and worried about my problems."

a. "I cannot recall why I'm living in this town." The patient in a fugue state frequently relocates and assumes a new identity while not recalling previous identity or places previously inhabited. The distracters are more consistent with depersonalization disorder, generalized anxiety disorder, or dissociative identity disorder. See relationship to audience response question.

10. A young child is found wandering alone at a mall. A male store employee approaches and asks where her parents are. She responds, "I don't know. Maybe you will take me home with you?" This sort of response in children may be due to: a. A lack of bonding as an infant b. A healthy confidence in the child c. Adequate parental bonding d. Normal parenting

a. A lack of bonding as an infant

Which scenario demonstrates a dissociative fugue? a. After being caught in an extramarital affair, a man disappeared but then reappeared months later with no memory of what occurred while he was missing. b. A man is extremely anxious about his problems and sometimes experiences dazed periods of several minutes passing without conscious awareness of them. c. A woman finds unfamiliar clothes in her closet, is recognized when she goes to new restaurants, and complains of "blackouts" despite not drinking. d. A woman reports that when she feels tired or stressed, it seems like her body is not real and is somehow growing smaller.

a. After being caught in an extramarital affair, a man disappeared but then reappeared months later with no memory of what occurred while he was missing. The patient in a dissociative fugue state relocates and lacks recall of his life before the fugue began. Often fugue states follow traumatic experiences and sometimes involve assuming a new identity. Such persons at some point find themselves in their new surroundings, unable to recall who they are or how they got there. A feeling of detachment from ones body or from the external reality is an indication of depersonalization disorder. Losing track of several minutes when highly anxious is not an indication of a dissociative disorder and is common in states of elevated anxiety. Finding evidence of having bought clothes or gone to restaurants without any explanation for these is suggestive of dissociative identity disorder, particularly when periods are lost to the patient (blackouts). See relationship to audience response question.

1. Which individuals are most at risk for displaying aggressive behavior? Select all that apply. a. An adolescent embarrassed in front of friends. b. A young male who feels rejected by the social group. c. A young adult depressed after the death of a friend. d. A middle-aged adult who feels that concerns are going unheard. e. A patient who was discovered telling a lie.

a. An adolescent embarrassed in front of friends. b. A young male who feels rejected by the social group. d. A middle-aged adult who feels that concerns are going unheard. e. A patient who was discovered telling a lie.

9. Twenty-four-hour observation is a good choice for restraint in which of the following patients? a. An inmate with suicidal ideation on hospice care b. A sex offender in the psychiatric intensive care unit c. An aggressive female with antisocial personality disorder d. An inmate diagnosed with paranoid schizophrenia

a. An inmate with suicidal ideation on hospice care

3. Which intervention(s) should the nurse implement when helping a patient expresses anger in an inappropriate manner? Select all that apply. a. Approach the patient in a calm, reassuring manner. b. Provide suggestions regarding acceptable ways of communicating anger. c. Warn the patient that being angry is not a healthy emotional state. d. Set limits on the angry behavior that will be tolerated. e. Allow any expression of anger as long as no one is hurt.

a. Approach the patient in a calm, reassuring manner. b. Provide suggestions regarding acceptable ways of communicating anger. d. Set limits on the angry behavior that will be tolerated.

4. Which guidelines should direct nursing care when deescalating an angry patient? Select all that apply. a. Intervene as quickly as possible b. Identify the trigger for the anger c. Behave calmly and respectfully d. Recognize the patient's need for increased personal space e. Demands are agreed to as long as they won't result in harm to anyone

a. Intervene as quickly as possible b. Identify the trigger for the anger c. Behave calmly and respectfully d. Recognize the patient's need for increased personal space

6. A pregnant woman is in a relationship with a male who routinely abuses her. Her unborn child may engage in high-risk behavior as a teen as a result of: a. Maternal stress b. Parental nurturing c. Appropriate stress responses in the brain d. Memories of the abuse

a. Maternal stress

A soldier in a combat zone tells the nurse, "I saw a child get blown up over a year ago, and I still keep seeing bits of flesh everywhere. I see something red, and the visions race back to my mind." Which phenomenon associated with posttraumatic stress disorder (PTSD) is the soldier describing? a. Reexperiencing b. Hyperarousal c. Avoidance d. Psychosis

a. Reexperiencing Spontaneous or cued recurrent, involuntary, and intrusive distressing memories of the traumatic events are often associated with PTSD. The soldier has described intrusive thoughts and visions associated with reexperiencing the traumatic event. This description does not indicate psychosis, hypervigilance, or avoidance.

A wife received news that her husband died of heart failure and called her family to come to the hospital. She angrily tells the nurse who cared for him, "He would still be alive if you had given him your undivided attention." Select the nurse's best intervention. a. Say to the wife, "I understand you are feeling upset. I will stay with you until your family comes." b. Say to the wife, "Your husband's heart was so severely damaged that it could no longer pump." c. Say to the wife, "I will call the health care provider to discuss this matter with you." d. Hold the wife's hand in silence until the family arrives.

a. Say to the wife, "I understand you are feeling upset. I will stay with you until your family comes." The nurse builds trust and shows compassion in the face of adjustment disorders. Therapeutic responses provide comfort. The nurse should show patience and tact while offering sympathy and warmth. The distracters are defensive, evasive, or placating.

Relaxation techniques help patients who have experienced major traumas because they: a. engage the parasympathetic nervous system. b. increase sympathetic stimulation. c. increase the metabolic rate. d. release hormones.

a. engage the parasympathetic nervous system. In response to trauma, the sympathetic arousal symptoms of rapid heart rate and rapid respiration prepare the person for flight or fight responses. Afterward, the dorsal vagal response damps down the sympathetic nervous system. This is a parasympathetic response with the heart rate and respiration slowing down and decreasing the blood pressure. Relaxation techniques promote activity of the parasympathetic nervous system.

The nurse who is counseling a patient with dissociative identity disorder should understand that the assessment of highest priority is: a. risk for self-harm. b. cognitive function. c. memory impairment. d. condition of self-esteem.

a. risk for self-harm. Assessments that relate to patient safety take priority. Patients with dissociative disorders may be at risk for suicide or self-mutilation, so the nurse must be alert for indicators of risk for self-injury. The other options are important assessments but rank below safety. Treatment motivation, while an important consideration, is not necessarily a part of the nursing assessment.

7. A nurse named Darryl has been hired to work in a psychiatric intensive care unit. He has undergone training on recognizing escalating anger. Which statement indicates that he understands danger signs in regard to aggression? a. "I need to be aware of patients who are withdrawn and sitting alone." b. "An obvious change in behavior is a risk factor for aggression." c. "Patients who seek constant attention are more likely to be violent." d. "Patients who talk to themselves are the most dangerous."

b. "An obvious change in behavior is a risk factor for aggression."

After the sudden death of his wife, a man says, "I can't live without her...she was my whole life." Select the nurse's most therapeutic reply. a. "Each day will get a little better." b. "Her death is a terrible loss for you." c. "It's important to recognize that she is no longer suffering." d. "Your friends will help you cope with this change in your life."

b. "Her death is a terrible loss for you." Adjustment disorders may be associated with grief. A statement that validates a bereaved persons loss is more helpful than false reassurances and clichs. It signifies understanding.

Two weeks ago, a soldier returned to the U.S. from active duty in a combat zone in Afghanistan. The soldier was diagnosed with posttraumatic stress disorder (PTSD). Which comment by the soldier requires the nurse's immediate attention? a. "It's good to be home. I missed my home, family, and friends." b. "I saw my best friend get killed by a roadside bomb. I don't understand why it wasn't me." c. "Sometimes I think I hear bombs exploding, but it's just the noise of traffic in my hometown." d. "I want to continue my education, but I'm not sure how I will fit in with other college students."

b. "I saw my best friend get killed by a roadside bomb. I don't understand why it wasn't me." The correct response indicates the soldier is thinking about death and feeling survivors guilt. These emotions may accompany suicidal ideation, which warrants the nurses follow-up assessment. Suicide is a high risk among military personnel diagnosed with posttraumatic stress disorder. One distracter indicates flashbacks, common with persons with PTSD, but not solely indicative that further problems exist. The other distracters are normal emotions associated with returning home and change.

2. A newly admitted male patient has a long history of aggressive behavior toward staff. Which statement by the nurse demonstrates the need for more information about the use of restraint? a. "If his behavior warrants restraints, someone will stay with him the entire time he's restrained." b. "I'll call the primary provider and get an as needed (prn) seclusion/restraint order." c. "If he is restrained, be sure he is offered food and fluids regularly." d. "Remember that physical restraints are our last resort."

b. "I'll call the primary provider and get an as needed (prn) seclusion/restraint order."

The unlicensed assistive personnel (UAP) says to the nurse, "That patient with amnesia looks fine, but when I talk to her, she seems vague. What should I be doing for her?" Select the nurse's best reply. a. "Spend as much time with her as you can and ask questions about her life." b. "Use short, simple sentences and keep the environment calm and protective." c. "Provide more information about her past to reduce the mysteries that are causing anxiety." d. "Structure her time with activities to keep her busy, stimulated, and regaining concentration."

b. "Use short, simple sentences and keep the environment calm and protective." Disruptions in ability to perform activities of daily living, confusion, and anxiety are often apparent in patients with amnesia. Offering simple directions to promote activities of daily living and reduce confusion helps increase feelings of safety and security. A calm, secure, predictable, protective environment is also helpful when a person is dealing with a great deal of uncertainty. Recollection of memories should proceed at its own pace, and the patient should only gradually be given information about her past. Asking questions that require recall that the patient does not possess will only add frustration. Quiet, undemanding activities should be provided as the patient tolerates them and should be balanced with rest periods; the patients time should not be loaded with demanding or stimulating activities.

The care plan for a patient diagnosed with a somatic disorder includes the nursing diagnosis Ineffective Coping. Which nursing diagnosis demonstrates a successful outcome for that diagnosis? A. Showers and dresses in clean clothes daily. B. Calls a friend to talk when feeling lonely C. Spends more time talking about pain in her abdomen D. Maintains focus and concentration

b. Calls a friend to talk when feeling lonely

A patient diagnosed with depersonalization disorder tells the nurse, "It's starting again. I feel as though I'm going to float away." Which intervention would be most appropriate at this point? a. Notify the health care provider of this change in the patient's behavior. b. Engage the patient in a physical activity such as exercise. c. Isolate the patient until the sensation has diminished. d. Administer a PRN dose of anti-anxiety medication.

b. Engage the patient in a physical activity such as exercise. Helping the patient apply a grounding technique, such as exercise, assists the patient to interrupt the dissociative process. Medication can help reduce anxiety but does not directly interrupt the dissociative process. Isolation would allow the sensation to overpower the patient. It is not necessary to notify the health care provider.

A nurse works with a patient diagnosed with posttraumatic stress disorder who has frequent flashbacks as well as persistent symptoms of arousal. Which intervention should be included in the plan of care? a. Trigger flashbacks intentionally in order to help the patient learn to cope with them. b. Explain that the physical symptoms are related to the psychological state. c. Encourage repression of memories associated with the traumatic event. d. Support "numbing" as a temporary way to manage intolerable feelings.

b. Explain that the physical symptoms are related to the psychological state. Persons with posttraumatic stress disorder often experience somatic symptoms or sympathetic nervous system arousal that can be confusing and distressing. Explaining that these are the bodys responses to psychological trauma helps the patient understand how such symptoms are part of the illness and something that will respond to treatment. This decreases powerlessness over the symptoms and helps instill a sense of hope. It also helps the patient to understand how relaxation, breathing exercises, and imagery can be helpful in symptom reduction. The goal of treatment for posttraumatic stress disorder is to come to terms with the event so treatment efforts would not include repression of memories or numbing. Triggering flashbacks would increase patient distress.

A soldier returned home from active duty in a combat zone in Afghanistan and was diagnosed with posttraumatic stress disorder (PTSD). The soldier says, "If there's a loud noise at night, I get under my bed because I think we're getting bombed." What type of experience has the soldier described? a. Illusion b. Flashback c. Nightmare d. Auditory hallucination

b. Flashback Flashbacks are dissociative reactions in which an individual feels or acts as if the traumatic event were recurring. Illusions are misinterpretations of stimuli, and although the experience is similar, it is better termed a flashback because of the diagnosis of PTSD. Auditory hallucinations have no external stimuli. Nightmares commonly accompany PTSD, but this experience was stimulated by an actual environmental sound.

The gas pedal on a person's car stuck on a busy interstate highway, causing the car to accelerate rapidly. For 20 minutes, the car was very difficult to control. Afterward, this person's cortisol regulation was compromised. Which assessment finding would the nurse expect associated with the dysregulation of cortisol? a. Weight gain b. Flashbacks c. Headache d. Diuresis

b. Flashbacks Cortisol is a hormone released in response to stress. Severe dissociation or mindflight occurs for those who have suffered significant trauma. The episodic failure of dissociation causes intrusive symptoms such as flashbacks, thus dysregulating cortisol. The cortisol level may go up or down, so diuresis and/or weight gain may or may not occur. Answering this question correctly requires that the student apply prior learning regarding the effects of cortisol.

3. The care plan of a male patient diagnosed with a dissociative disorder includes the nursing diagnosis ineffective coping. Which behavior demonstrated by the patient supports this nursing diagnosis? a. Has no memory of the physical abuse he endured. b. Using both alcohol and marijuana. c. Often reports being unaware of surroundings. d. Reports feelings of "not really being here."

b. Using both alcohol and marijuana.

7. Maggie, a child in protective custody, is found to have an imaginary friend, Holly. Her foster family shares this information with the nurse. The nurse teaches the family members about children who have suffered trauma and knows her teaching was effective when the foster mother states: a. "I understand that imaginary friends are abnormal." b. "I understand that imaginary friends are a maladaptive behavior." c. "I understand that imaginary friends are a coping mechanism." d. "I understand that we should tell the child that imaginary friends are unacceptable."

c. "I understand that imaginary friends are a coping mechanism."

9. During a routine health screening, a grieving widow whose husband died 15 months ago reports emptiness, a loss of self, difficulty thinking of the future, and anger at her dead husband. The nurse suggests bereavement counseling. The widow is most likely suffering from: a. Major depression b. Normal grieving c. Adjustment disorder d. Posttraumatic stress disorder

c. Adjustment disorder

A woman just received notification that her husband died. She approaches the nurse who cared for him during his last hours and says angrily, "If you had given him your undivided attention, he would still be alive." How should the nurse analyze this behavior? a. The comment suggests potential allegations of malpractice. b. In some cultures, grief is expressed solely through anger. c. Anger is an expected emotion in an adjustment disorder. d. The patient had ambivalent feelings about her husband.

c. Anger is an expected emotion in an adjustment disorder. Symptoms of adjustment disorder run the gamut of all forms of distress including guilt, depression, and anger. Anger may protect the bereaved from facing the devastating reality of loss.

A soldier who served in a combat zone returned to the U.S. The soldier's spouse complains to the nurse, "We had planned to start a family, but now he won't talk about it. He won't even look at children." The spouse is describing which symptom associated with posttraumatic stress disorder (PTSD)? a. Reexperiencing b. Hyperarousal c. Avoidance d. Psychosis

c. Avoidance Physiological reactions to reminders of the event that include persistent avoidance of stimuli associated with the trauma results in the individuals avoiding talking about the event or avoiding activities, people, or places that arouse memories of the trauma. Avoidance is exemplified by a sense of foreshortened future and estrangement. There is no evidence this soldier is having hyperarousal or reexperiencing war-related traumas. Psychosis is not evident.

8. An effective method of preventing escalation in an environment with violent offenders is to develop a level of trust through: a. A casual authoritative demeanor b. Keeping patients busy c. Brief, frequent, nonthreatening encounters d. Threats of seclusion or punishment

c. Brief, frequent, nonthreatening encounters

A patient states, "I feel detached and weird all the time. It is as though I am looking at life through a cloudy window. Everything seems unreal. It really messes up things at work and school." This scenario is most suggestive of which health problem? a. Acute stress disorder b. Dissociative amnesia c. Depersonalization disorder d. Disinhibited social engagement disorder

c. Depersonalization disorder Depersonalization disorder involves a persistent or recurrent experience of feeling detached from and outside oneself. Although reality testing is intact, the experience causes significant impairment in social or occupational functioning and distress to the individual. Dissociative amnesia involves memory loss. Children with disinhibited social engagement disorder demonstrate no normal fear of strangers and are unusually willing to go off with strangers. Individuals with ASD experience three or more dissociative symptoms associated with a traumatic event, such as a subjective sense of numbing, detachment, or absence of emotional responsiveness; a reduction in awareness of surroundings; derealization; depersonalization or dissociative amnesia. In the scenario, the patient experiences only one symptom.

4. Which statement accurately describes the effects of emotional trauma on the individual physically? a. Emotional trauma is a distinct category and unrelated to physical problems b. The physical manifestations of emotional trauma are usually temporary c. Emotional trauma is often manifested as physical symptoms d. Patients are more aware of the physical problems caused by trauma

c. Emotional trauma is often manifested as physical symptoms

2. Which goal should be addressed initially when providing care for 10-year-old Harper who is diagnosed with posttraumatic stress disorder (PTSD)? a. Harper will be able to identify feelings through the use of play therapy. b. Harper and her parents will have access to protective resources available through social services. c. Harper will demonstrate the effective use of relaxation techniques to restore a sense of control over disturbing thoughts. d. Harper and her parents will demonstrate an understanding of the personal human response to traumatic events.

c. Harper will demonstrate the effective use of relaxation techniques to restore a sense of control over disturbing thoughts.

A person runs from a crowded nightclub after a pyrotechnics show causes the building to catch fire. Which division of the autonomic nervous system will be stimulated in response to this experience? a. Limbic system b. Peripheral nervous system c. Sympathetic nervous system d. Parasympathetic nervous system

c. Sympathetic nervous system The autonomic nervous system is comprised of the sympathetic (fight or flight response) and parasympathetic nervous system (relaxation response). In times of stress, the sympathetic nervous system is stimulated. A person would experience stress associated with the experience of being in danger. The peripheral nervous system responds to messages from the sympathetic nervous system. The limbic system processes emotional responses but is not specifically part of the autonomic nervous system.

A soldier returns to the United States from active duty in a combat zone in Afghanistan. The soldier is diagnosed with posttraumatic stress disorder (PTSD). The nurse's highest priority is to screen this soldier for: a. bipolar disorder. b. schizophrenia. c. depression. d. dementia.

c. depression. Comorbidities for adults with PTSD include depression, anxiety disorders, sleep disorders, and dissociative disorders. Incidence of the disorders identified in the distracters is similar to the general population.

After major reconstructive surgery, a patient's wounds dehisced. Extensive wound care was required for 6 months, causing the patient to miss work and social activities. Which pathophysiology would be expected for this patient? Dysfunction of the: a. pons. b. occipital lobe. c. hippocampus. d. hypothalamus.

c. hippocampus. The scenario presents chronic and potentially debilitating stress. If arousal continues unabated, neuronal changes occur that alter the neural circuitry of the prefrontal cortex, reducing the size the hippocampus so that memory is impaired.

A soldier returned home last year after deployment to a war zone. The soldier's spouse complains, "We were going to start a family, but now he won't talk about it. He will not look at children. I wonder if we're going to make it as a couple." Select the nurse's best response. a. "Posttraumatic stress disorder often changes a person's sexual functioning." b. "I encourage you to continue to participate in social activities where children are present." c. "Have you talked with your spouse about these reactions? Sometimes we just need to confront behavior." d. "Posttraumatic stress disorder often strains relationships. Here are some community resources for help and support."

d. "Posttraumatic stress disorder often strains relationships. Here are some community resources for help and support." Posttraumatic stress disorder precipitates changes that often lead to divorce. Its important to provide support to both the veteran and spouse. Confrontation will not be effective. While its important to provide information, on-going support will be more effective.

A store clerk was killed during a robbery 2 weeks ago. His widow, who has a long history of schizoaffective disorder, cries spontaneously when talking about his death. Select the nurse's most therapeutic response. a. "Are you taking your medications the way they are prescribed?" b. "This loss is harder to accept because of your mental illness. Do you think you should be hospitalized?" c. "I'm worried about how much you are crying. Your grief over your husband's death has gone on too long." d. "The unexpected death of your husband is very painful. I'm glad you are able to talk about your feelings."

d. "The unexpected death of your husband is very painful. I'm glad you are able to talk about your feelings." The patient is expressing feelings related to the loss, and this is an expected and healthy behavior. This patient is at risk for a maladaptive response because of the history of a serious mental illness, but the nurses priority intervention is to form a therapeutic alliance and support the patients expression of feelings. Crying at 2 weeks after his death is expected and normal.

8. An incest survivor undergoing treatment at the mental health clinic is relieved when she learns that her anxiety and depression are: a. Going to be eradicated with treatment b. Normal and will soon pass c. Abnormal but will pass d. A normal reaction to posttraumatic events

d. A normal reaction to posttraumatic events

5. The school nurse has been alerted to the fact that an 8-year-old boy routinely playacts as a police officer "locking up" other children on the playground to the point where the children get scared. The nurse recognizes that this behavior is most likely an indication of: a. The need to dominate others b. Inventing traumatic events c. A need to develop close relationships d. A potential symptom of traumatization

d. A potential symptom of traumatization

5. Which comorbid condition would result in cautious use of a selective serotonin reuptake inhibitors for a patient with chronic aggression? a. Asthma b. Anxiety disorder c. Glaucoma d. Bipolar disorder

d. Bipolar disorder

10. Chronic obstructive pulmonary disease, spinal injury, seizure disorder, and pregnancy are conditions that: a. Frequently result in out of control behavior. b. Respond well to therapeutic holding. c. Necessitate the use of only two-point restraint. d. Contraindicate restraint and seclusion.

d. Contraindicate restraint and seclusion.

A soldier returned 3 months ago from Afghanistan and was diagnosed with posttraumatic stress disorder (PTSD). Which social event would be most disturbing for this soldier? a. Halloween festival with neighborhood children b. Singing carols around a Christmas tree c. A family outing to the seashore d. Fireworks display on July 4th

d. Fireworks display on July 4th The exploding noises associated with fireworks are likely to provoke exaggerated responses for this soldier. The distracters are not associated with offensive sounds.

A soldier served in combat zones in Iraq during 2010 and was deployed to Afghanistan in 2013. When is it most important for the nurse to screen for signs and symptoms of posttraumatic stress disorder (PTSD)? a. Immediately upon return to the U.S. from Afghanistan b. Before departing Afghanistan to return to the U.S. c. One year after returning from Afghanistan d. Screening should be on-going

d. Screening should be on-going PTSD can have a very long lag time, months to years. Screening should be on-going.

Select the correct etiology to complete this nursing diagnosis for a patient with dissociative identity disorder. Disturbed personal identity related to: a. obsessive fears of harming self or others. b. poor impulse control and lack of self-confidence. c. depressed mood secondary to nightmares and intrusive thoughts. d. cognitive distortions associated with unresolved childhood abuse issues.

d. cognitive distortions associated with unresolved childhood abuse issues. Nearly all patients with dissociative identity disorder have a history of childhood abuse or trauma. None of the other etiology statements is relevant. See relationship to audience response question.

Four teenagers died in an automobile accident. One week later, which behavior by the parents of these teenagers most clearly demonstrates resilience? The parents who: a. visit their teenager's grave daily. b. return immediately to employment. c. discuss the accident within the family only. d. create a scholarship fund at their child's high school.

d. create a scholarship fund at their child's high school. Resilience refers to positive adaptation or the ability to maintain or regain mental health despite adversity. Loss of a child is among the highest-risk situations for maladaptive grieving. The parents who create a scholarship fund are openly expressing their feelings and memorializing their child. The other parents in this question are isolating themselves and/or denying their feelings. Visiting the grave daily shows active continued mourning but is not as strongly indicative of resilience as the correct response.

A child drowned while swimming in a local lake 2 years ago. Which behavior indicates the child's parents have adapted to their loss? The parents: a. visit their child's grave daily. b. maintain their child's room as the child left it 2 years ago. c. keep a place set for the dead child at the family dinner table. d. throw flowers on the lake at each anniversary date of the accident.

d. throw flowers on the lake at each anniversary date of the accident. Resilience refers to positive adaptation or the ability to maintain or regain mental health despite adversity. Loss of a child is among the highest-risk situations for an adjustment disorder and maladaptive grieving. The parents who throw flowers on the lake on each anniversary date of the accident are openly expressing their feelings. The other behaviors are maladaptive because of isolating themselves and/or denying their feelings. After 2 years, the frequency of visiting the grave should have decreased.


संबंधित स्टडी सेट्स

ECO 155 Chapter 16 (Final Review)

View Set

foundations of entrepreneurship exam #1

View Set

3.5 Profitability and liquidity ratio analysis!

View Set

PrepU Chapter 42: Assessment and Management of Patients with Obesity

View Set

Advanced Computer Science Chapters 1-8 Multiple Choice

View Set

Med Surg I Prep U Chapter 37: Assessment and Management of Patients With Allergic Disorders

View Set